Difference between revisions of "2005 IMO Problems/Problem 3"

 
Line 1: Line 1:
"""
+
\documentclass{article}
Solution:
+
\usepackage{amsmath}
 +
\usepackage{amssymb}
 +
\begin{document}
 +
 
 +
\section*{Problem}
 +
Let
 +
x
 +
,
 +
y
 +
,
 +
z
 +
x,y,z be positive real numbers such that
 +
x
 +
y
 +
z
 +
 +
1
 +
xyz≥1. Prove that:
  
First we can easily see
 
 
x
 
x
 
5
 
5
Line 16: Line 32:
 
z
 
z
 
2
 
2
+
+
x
+
y
4
+
5
 
 
x
 
x
 
4
 
+
 
 
y
 
y
 
2
 
2
 +
y
 +
5
 
+
 
+
 
z
 
z
 
2
 
2
x  
+
+
 +
x
 +
2
 +
+
 +
z
 
5
 
5
+y
+
 +
z
 
2
 
2
  +z  
+
z
 +
5
 +
+
 +
x
 +
2
 +
+
 +
y
 +
2
 +
 +
0.
 +
x
 +
5
 +
+y
 +
2
 +
  +z  
 
2
 
2
 
   
 
   
Line 42: Line 75:
 
   
 
   
 
 
 
+
  +
x
+
y
4
+
5
  +y
+
+z
 +
2
 +
  +x
 
2
 
2
  +z
+
   
 +
y
 +
5
 +
−y
 
2
 
2
 
   
 
   
x
 
4
 
−x
 
 
 
 
+
+
+
z
x
+
5
(
+
+x  
x
+
2
+
+y
1
 
)
 
 
2
 
2
(
+
x
+
z
 +
5
 +
−z
 
2
 
2
+
+
 +
 +
≥0.
 +
\section*{Solution}
 +
We first show that for any positive real numbers
 
x
 
x
+
+
,
1
 
)
 
(
 
 
y
 
y
2
+
,
+
 
 
z
 
z
 +
x,y,z, we have
 +
 +
x
 +
5
 +
 +
x
 
2
 
2
)
 
(
 
 
x
 
x
 
5
 
5
Line 87: Line 126:
 
z
 
z
 
2
 
2
)
+
(
+
x
 +
4
 +
 +
x
 
x
 
x
 
4
 
4
Line 97: Line 139:
 
z
 
z
 
2
 
2
)
+
.
+
x  
0
 
 
(x  
 
 
5
 
5
 
  +y  
 
  +y  
Line 107: Line 146:
 
  +z  
 
  +z  
 
2
 
2
  )(x  
+
   
 +
x
 +
5
 +
−x
 +
2
 +
 +
 +
 +
x  
 
4
 
4
 
  +y  
 
  +y  
Line 113: Line 160:
 
  +z  
 
  +z  
 
2
 
2
  )
+
   
x(x−1)
+
x  
2
+
4
  (x
+
  −x
2
 
+x+1)(y
 
2
 
+z
 
2
 
)
 
 
 
  ≥0 which is true for
+
  .
x
+
Indeed, consider the difference:
,
+
\begin{align*}
y
+
&\frac{x^5 - x^2}{x^5 + y^2 + z^2} - \frac{x^4 - x}{x^4 + y^2 + z^2} \
,
+
&= \frac{(x^5 - x^2)(x^4 + y^2 + z^2) - (x^4 - x)(x^5 + y^2 + z^2)}{(x^5 + y^2 + z^2)(x^4 + y^2 + z^2)}.
z
+
\end{align*}
x,y,z and
+
The numerator simplifies as follows:
z
+
\begin{align*}
z are positive real numbers.
+
&(x^5 - x^2)(x^4 + y^2 + z^2) - (x^4 - x)(x^5 + y^2 + z^2) \
 +
&= x^5x^4 + x^5(y^2+z^2) - x^2x^4 - x^2(y^2+z^2) - x^4x^5 - x^4(y^2+z^2) + x x^5 + x(y^2+z^2) \
 +
&= x^5(y^2+z^2) - x^2(y^2+z^2) - x^4(y^2+z^2) + x(y^2+z^2) \
 +
&= (y^2+z^2)(x^5 - x^4 - x^2 + x) \
 +
&= (y^2+z^2)x(x^4 - x^3 - x + 1) \quad \text{(but wait, check: actually, } x^5 - x^4 - x^2 + x = x(x^4 - x^3 - x + 1) \text{? That doesn't factor nicely)}.
 +
\end{align*}
 +
Alternatively, note that:
  
It's enough to prove:
 
 
x
 
x
4
+
5
 
 
 
x
 
x
 +
2
 +
=
 
x
 
x
4
 
+
 
y
 
 
2
 
2
+
+
(
z
+
x
2
+
3
+
 
y
 
4
 
 
 
y
+
1
y
+
)
4
+
=
+
+
x
z
 
 
2
 
2
+
+
(
 +
x
 +
 +
1
 +
)
 +
(
 +
x
 +
2
 +
+
 +
x
 +
+
 +
1
 +
)
 +
,
 
x
 
x
2
 
+
 
z
 
 
4
 
4
 
 
z
+
x
z
+
=
4
+
x
 +
(
 +
x
 +
3
 +
 +
1
 +
)
 +
=
 +
x
 +
(
 +
x
 +
 +
1
 +
)
 +
(
 +
x
 +
2
 
+
 
+
 
x
 
x
2
 
 
+
 
+
y
+
1
2
+
)
+
.
0
 
 
x  
 
x  
4
+
5
  +y
+
  −x
 
2
 
2
  +z
+
  =x
 
2
 
2
   
+
  (x  
x  
+
3
4
+
  −1)=x
  −x
 
 
+
 
y
 
4
 
+z
 
 
2
 
2
  +x  
+
  (x−1)(x  
 
2
 
2
   
+
  +x+1),x
y
 
 
4
 
4
  −y
+
  −x=x(x
+
3
  +
+
  −1)=x(x−1)(x  
z
 
4
 
+x  
 
 
2
 
2
  +y
+
  +x+1).
2
+
So then:
+
\begin{align*}
z  
+
&\frac{x^5 - x^2}{x^5 + y^2 + z^2} - \frac{x^4 - x}{x^4 + y^2 + z^2} \
4
+
&= \frac{x^2(x-1)(x^2+x+1)}{x^5 + y^2 + z^2} - \frac{x(x-1)(x^2+x+1)}{x^4 + y^2 + z^2} \
−z
+
&= x(x-1)(x^2+x+1) \left( \frac{x}{x^5 + y^2 + z^2} - \frac{1}{x^4 + y^2 + z^2} \right).
+
\end{align*}
≥0
+
Now, combine the terms inside the parentheses:
  
Or
 
 
x
 
x
4
 
 
x
 
x
4
+
5
 
+
 
+
 
y
 
y
Line 223: Line 276:
 
z
 
z
 
2
 
2
 +
 +
1
 +
x
 +
4
 
+
 
+
 
y
 
y
4
+
2
y
 
4
 
 
+
 
+
 
z
 
z
 
2
 
2
+
+
=
 +
x
 +
(
 
x
 
x
2
 
+
 
z
 
4
 
z
 
 
4
 
4
 
+
 
+
x
+
y
 
2
 
2
 
+
 
+
y
+
z
 
2
 
2
+
)
 +
 +
(
 
x
 
x
x
+
5
4
 
 
+
 
+
 
y
 
y
Line 255: Line 308:
 
z
 
z
 
2
 
2
 +
)
 +
(
 +
x
 +
5
 
+
 
+
 
y
 
y
 +
2
 +
+
 +
z
 +
2
 +
)
 +
(
 +
x
 +
4
 +
+
 
y
 
y
4
+
2
 
+
 
+
 
z
 
z
 
2
 
2
 +
)
 +
=
 +
(
 +
x
 +
5
 
+
 
+
 
x
 
x
 +
(
 +
y
 
2
 
2
 
+
 
+
 
z
 
z
z
+
2
4
+
)
+
+
)
 +
 +
(
 
x
 
x
2
+
5
 
+
 
+
 
y
 
y
 
2
 
2
x
+
+
4
+
z
+y
 
 
2
 
2
+z
+
)
 +
(
 +
x
 +
5
 +
+
 +
y
 
2
 
2
+
+
x
+
z
4
 
 
 
+  
 
y
 
4
 
+z  
 
 
2
 
2
+x
+
)
2
+
(
+
x
y
 
 
4
 
4
+
+
+
y
+  
 
z
 
4
 
+x
 
 
2
 
2
+y
+
+
 +
z
 
2
 
2
+
)
z
+
=
4
+
(
+
x
+
+
1
x
+
)
4
+
(
+y  
+
y
 
2
 
2
+z  
+
+
 +
z
 
2
 
2
+
)
 +
(
 
x
 
x
+
5
+  
+
+
y  
+
y
4
 
+z
 
 
2
 
2
+x
+
+
 +
z
 
2
 
2
+
)
y
+
(
+
x
+
 
z
 
 
4
 
4
+x
+
+
 +
y
 +
2
 +
+
 +
z
 
2
 
2
 +
)
 +
.
 +
x
 +
5
 
  +y  
 
  +y  
 +
2
 +
+z
 
2
 
2
 
   
 
   
z
+
x
 
 
 +
 +
x
 +
4
 +
+y
 +
2
 +
+z
 +
2
 
   
 
   
 
+
1
Let
+
t
+
=  
=
+
(x  
x
+
5
+
+
  +y  
y
 
+
 
z
 
 
t
 
 
3
 
(
 
x
 
y
 
z
 
)
 
1
 
/
 
3
 
 
3
 
t=x+y+z→t≥3(xyz)
 
1/3
 
  ≥3 and
 
x
 
y
 
+
 
y
 
z
 
+
 
z
 
x
 
 
t
 
 
2
 
2
3
+
+z
 
x
 
 
2
 
2
+
+
)(x
y
+
4
 +
+y  
 
2
 
2
+
+
+z  
z
 
 
2
 
2
xy+yz+zx≤
+
)
3
+
x(x
t
+
4
 +
+y
 
2
 
2
   
+
  +z
 
≤x
 
 
2
 
2
 +
)−(x
 +
5
 
  +y  
 
  +y  
 
2
 
2
 
  +z  
 
  +z  
 
2
 
2
   
+
  )
 
+
By C-S:
+
=
(
+
(x
x
+
5
 +
+y
 
2
 
2
+
+
+z
y
 
 
2
 
2
+
+
)(x
z
+
4
 +
+y
 
2
 
2
)
+
+z
 
2
 
2
+
)
(
+
(x
x
+
5
4
+
+x(y
+
+
2
y
+
+z
 +
2
 +
))−(x
 +
5
 +
+y  
 
2
 
2
+
+
+z  
z
 
 
2
 
2
)
+
)
(
+
1
+
=
+
+
(x
y
+
5
 +
+y  
 
2
 
2
+
+
+z  
z
 
 
2
 
2
)
+
)(x  
 
x
 
x
 
 
4
 
4
+
+
+y  
y
 
 
2
 
2
+
+
+z  
z
 
 
2
 
2
+
)
 +
(x−1)(y
 +
2
 +
+z
 +
2
 +
)
 +
 +
.
 +
Thus, the entire difference becomes:
 +
 
 
x
 
x
 
(
 
(
 +
x
 +
 
1
 
1
 +
)
 +
(
 +
x
 +
2
 
+
 
+
 +
x
 +
+
 +
1
 +
)
 +
 +
(
 +
x
 +
 +
1
 +
)
 +
(
 
y
 
y
 
2
 
2
Line 457: Line 535:
 
(
 
(
 
x
 
x
2
+
5
 
+
 
+
 
y
 
y
Line 465: Line 543:
 
2
 
2
 
)
 
)
 +
(
 +
x
 +
4
 +
+
 +
y
 
2
 
2
(x
+
+
 +
z
 
2
 
2
+y
+
)
 +
=
 +
x
 +
(
 +
x
 +
 +
1
 +
)
 
2
 
2
+z
+
(
 +
x
 
2
 
2
)  
+
+
 +
x
 +
+
 +
1
 +
)
 +
(
 +
y
 
2
 
2
≤(x
+
+
4
+
z
+y
 
 
2
 
2
+z
+
)
 +
(
 +
x
 +
5
 +
+
 +
y
 
2
 
2
)(1+y
+
+
 +
z
 
2
 
2
+z
+
)
2
+
(
)→
+
x
x  
 
 
4
 
4
 +
+
 +
y
 +
2
 +
+
 +
z
 +
2
 +
)
 +
 +
0.
 +
x(x−1)(x
 +
2
 +
+x+1)⋅
 +
(x
 +
5
 
  +y  
 
  +y  
 
2
 
2
 
  +z  
 
  +z  
 
2
 
2
   
+
  )(x  
x
+
4
 
 
(x  
 
2
 
 
  +y  
 
  +y  
 
2
 
2
 
  +z  
 
  +z  
 
2
 
2
  )  
+
  )
2
+
(x−1)(y  
 
x(1+y  
 
 
2
 
2
 
  +z  
 
  +z  
Line 510: Line 620:
 
  )
 
  )
 
 
   
+
  =
 
+
(x
+
5
⇒ RHS
+
+y
+
2
x
+
+z
(
+
2
1
+
)(x  
+
+
4
y
+
+y
 +
2
 +
+z
 +
2
 +
)
 +
x(x−1)
 +
2
 +
(x
 +
2
 +
+x+1)(y
 +
2
 +
+z
 
2
 
2
+
+
)
z
+
 +
≥0.
 +
Hence,
 +
 
 +
x
 +
5
 +
 +
x
 
2
 
2
)
+
x
 +
5
 
+
 
+
 
y
 
y
(
+
2
1
 
 
+
 
+
 
z
 
z
 
2
 
2
+
+
 
x
 
x
2
+
4
)
+
+
 
z
 
(
 
1
 
+
 
 
x
 
x
2
 
+
 
y
 
2
 
)
 
(
 
 
x
 
x
2
+
4
 
+
 
+
 
y
 
y
Line 556: Line 673:
 
z
 
z
 
2
 
2
)
+
.
 +
x
 +
5
 +
+y
 +
2
 +
+z
 +
2
 +
 +
x
 +
5
 +
−x
 
2
 
2
=
+
x
+
+
+
y
+
x  
+
+
4
z
+
+y
+
+
2
(
+
+z  
x
+
2
y
+
+
+
x  
 +
4
 +
−x
 +
 +
.
 +
Similarly, we have the analogous inequalities for
 
y
 
y
 +
y and
 
z
 
z
+
+
z. Therefore, it suffices to prove that
z
+
 
 
x
 
x
)
+
4
(
 
x
 
+
 
y
 
+
 
z
 
)
 
 
 
3
 
 
x
 
x
y
 
z
 
(
 
 
x
 
x
2
+
4
 
+
 
+
 
y
 
y
Line 596: Line 718:
 
z
 
z
 
2
 
2
)
+
+
 +
y
 +
4
 +
 +
y
 +
y
 +
4
 +
+
 +
z
 +
2
 +
+
 +
x
 +
2
 +
+
 +
z
 +
4
 +
 +
z
 +
z
 +
4
 +
+
 +
x
 
2
 
2
+
+
(x
+
y
 
2
 
2
 +
 +
0
 +
,
 +
x
 +
4
 
  +y  
 
  +y  
 
2
 
2
 
  +z  
 
  +z  
2
 
)
 
 
2
 
2
 
   
 
   
x(1+y  
+
x  
2
+
4
 +
−x
 +
 +
+  
 +
y  
 +
4
 
  +z  
 
  +z  
2
 
)+y(1+z
 
 
2
 
2
 
  +x  
 
  +x  
 
2
 
2
  )+z(1+x
+
   
2
+
y  
+y  
+
4
2
+
  −y
  )
 
 
 
  =
+
  +
(x  
+
z
 +
4
 +
+x  
 
2
 
2
 
  +y  
 
  +y  
2
 
+z
 
2
 
)
 
 
2
 
2
 
   
 
   
x+y+z+(xy+yz+zx)(x+y+z)−3xyz
+
z  
 +
4
 +
−z
 
 
   
+
  ≥0,
 +
or equivalently,
  
+
x
t
+
4
 +
x
 +
4
 
+
 
+
t
+
y
3
+
2
3
+
+
t
+
z
 +
2
 +
+
 +
y
 +
4
 +
y
 
4
 
4
=
 
3
 
t
 
3
 
 
+
 
+
9
+
z
t
+
2
+
+
27
+
x
t
+
2
 +
+
 +
z
 +
4
 +
z
 
4
 
4
+
+
1
+
x
+
2
(
+
+
t
+
y
+
2
3
 
)
 
(
 
t
 
3
 
 
9
 
)
 
 
 
0
+
x
 
t
 
4
 
 
t+
 
3
 
t
 
3
 
 
 
 
 
=
 
t
 
4
 
 
3t
 
3
 
+9t−27
 
 
≤1⇔(t−3)(t
 
3
 
−9)≥0
 
which is true for positive real numbers <math>x, y, z</math>.
 
 
 
It's enough to prove:
 
 
 
 
x
 
x
 
4
 
4
+
+
x
 
x
 
4
 
+
 
 
y
 
y
 
2
 
2
Line 711: Line 828:
 
2
 
2
 
+
 
+
y
 
4
 
 
 
y
 
y
 
y
 
y
Line 724: Line 838:
 
2
 
2
 
+
 
+
z
 
4
 
 
 
z
 
z
 
z
 
z
Line 736: Line 847:
 
y
 
y
 
2
 
2
+
.
0.
+
(1)
 
x  
 
x  
 
4
 
4
Line 747: Line 858:
 
x  
 
x  
 
4
 
4
  −x
+
   
 
 
 
  +  
 
  +  
Line 759: Line 870:
 
y  
 
y  
 
4
 
4
  −y
+
   
 
 
 
  +  
 
  +  
Line 771: Line 882:
 
z  
 
z  
 
4
 
4
  −z
+
   
 
 
  ≥0.
+
 
Or equivalently,
+
x  
 
 
x
 
 
4
 
4
 +
+y
 +
2
 +
+z
 +
2
 +
 
x
 
x
 +
 +
+
 +
y
 
4
 
4
+
+
+z
y
 
 
2
 
2
+
+
+x
z
 
 
2
 
2
+
+
 
y
 
y
 +
 +
+
 +
z
 
4
 
4
y
+
+x
4
 
+
 
z
 
 
2
 
2
+
+
+y
x
 
 
2
 
2
+
+
 
z
 
z
4
+
z
+
.(1)
4
+
Let
+
+
t
 +
=
 
x
 
x
2
 
 
+
 
+
 
y
 
y
2
+
+
+
z
 +
t=x+y+z. Since
 
x
 
x
 +
,
 +
y
 +
,
 +
z
 +
>
 +
0
 +
x,y,z>0 and
 
x
 
x
4
 
+
 
 
y
 
y
2
 
+
 
 
z
 
z
2
+
+
+
1
 +
xyz≥1, by AM–GM we have
 +
t
 +
 +
3
 +
x
 
y
 
y
 +
z
 +
3
 +
 +
3
 +
t≥3
 +
3
 +
 
 +
xyz
 +
 +
≥3. Also, note that
 +
x
 
y
 
y
4
 
 
+
 
+
 +
y
 
z
 
z
2
 
 
+
 
+
 +
z
 
x
 
x
 +
 +
t
 +
2
 +
3
 +
xy+yz+zx≤
 +
3
 +
t
 
2
 
2
+
+
z
+
z
+
  and
4
 
+
 
 
x
 
x
 
2
 
2
Line 838: Line 977:
 
y
 
y
 
2
 
2
.
+
+
 +
z
 +
2
 +
 +
t
 +
2
 +
3
 
x  
 
x  
4
+
2
 
  +y  
 
  +y  
 
2
 
2
 
  +z  
 
  +z  
 
2
 
2
   
+
 
x
+
3
4
+
t
 +
2
 
   
 
   
 
 
  +
+
  .
y
+
 
4
+
Now, we estimate the right-hand side of (1). By the Cauchy–Schwarz inequality, we have:
+z
+
 
 +
(
 +
x
 
2
 
2
+x
+
+
 +
y
 
2
 
2
+
+
y
+
z
4
+
2
+
)
+
2
+
+
z
+
(
 +
x
 
4
 
4
+x  
+
+
 +
y
 +
2
 +
+
 +
z
 +
2
 +
)
 +
(
 +
1
 +
+
 +
y
 +
2
 +
+
 +
z
 +
2
 +
)
 +
,
 +
(x  
 
2
 
2
 
  +y  
 
  +y  
 
2
 
2
   
+
  +z  
z  
+
2
4
+
  )
   
+
2
+
  ≤(x  
 
 
x  
 
 
4
 
4
 
  +y  
 
  +y  
Line 881: Line 1,046:
 
  +z  
 
  +z  
 
2
 
2
   
+
  )(1+y  
x
+
2
 
+  
 
y  
 
4
 
 
  +z  
 
  +z  
 
2
 
2
  +x
+
  ),
2
+
since
 
y
 
 
+
 
z
 
4
 
+x
 
2
 
+y
 
2
 
 
z
 
 
.
 
Let <math>t = x + y + z</math>. Then by AM–GM, <math>t \geq 3(xyz)^{1/3} \geq 3</math> and <math>xy + yz + zx \leq \frac{t^2}{3} \leq x^2 + y^2 + z^2</math>.
 
 
 
By Cauchy–Schwarz:
 
  
 
(
 
(
Line 920: Line 1,064:
 
)
 
)
 
2
 
2
+
=
 
(
 
(
 
x
 
x
4
+
2
 +
 +
1
 
+
 
+
 
y
 
y
2
+
 +
y
 
+
 
+
 
z
 
z
 +
 +
z
 +
)
 
2
 
2
)
+
 
(
 
(
1
+
x
 +
4
 
+
 
+
 
y
 
y
Line 940: Line 1,091:
 
2
 
2
 
)
 
)
+
(
x
+
1
x
+
2
4
 
 
+
 
+
 
y
 
y
Line 950: Line 1,100:
 
z
 
z
 
2
 
2
+
)
 +
=
 +
(
 
x
 
x
(
+
4
1
 
 
+
 
+
 
y
 
y
Line 962: Line 1,113:
 
)
 
)
 
(
 
(
x
+
1
2
 
 
+
 
+
 
y
 
y
Line 971: Line 1,121:
 
2
 
2
 
)
 
)
2
 
 
.
 
.
 
(x  
 
(x  
Line 980: Line 1,129:
 
2
 
2
 
  )  
 
  )  
 +
2
 +
=(x
 +
2
 +
⋅1+y⋅y+z⋅z)
 
2
 
2
 
  ≤(x  
 
  ≤(x  
Line 987: Line 1,140:
 
  +z  
 
  +z  
 
2
 
2
  )(1+y  
+
  )(1  
 +
2
 +
+y  
 
2
 
2
 
  +z  
 
  +z  
 
2
 
2
  )
+
  )=(x  
x  
 
 
4
 
4
 
  +y  
 
  +y  
Line 998: Line 1,152:
 
  +z  
 
  +z  
 
2
 
2
   
+
  )(1+y
x
 
 
 
(x
 
 
2
 
2
  +y  
+
  +z
 +
2
 +
).
 +
It follows that
 +
 
 +
1
 +
x
 +
4
 +
+
 +
y
 +
2
 +
+
 +
z
 
2
 
2
+z
+
 +
1
 +
+
 +
y
 
2
 
2
)
+
+
 +
z
 
2
 
2
+
(
x(1+y
+
x
 
2
 
2
+z
+
+
 +
y
 
2
 
2
)
 
 
.
 
Hence, the right-hand side (RHS) satisfies
 
\begin{align*}
 
\text{RHS} &\leq \frac{x(1 + y^2 + z^2) + y(1 + z^2 + x^2) + z(1 + x^2 + y^2)}{(x^2 + y^2 + z^2)^2} \
 
&= \frac{x+y+z + (xy+yz+zx)(x+y+z) - 3xyz}{(x^2 + y^2 + z^2)^2}.
 
\end{align*}
 
Since <math>x^2+y^2+z^2 \geq \frac{t^2}{3}</math> and <math>xy+yz+zx \leq \frac{t^2}{3}</math>, we have
 
 
RHS
 
 
t
 
 
+
 
+
t
+
z
3
+
2
3
 
t
 
4
 
/
 
9
 
=
 
9
 
t
 
4
 
(
 
t
 
+
 
t
 
3
 
3
 
 
)
 
)
=
+
2
9
+
,
t
+
x
 
4
 
4
+
+y
3
+
2
t
+
+z
+
+
2
t
+
3
+
1
3
+
=
+
3
+
(x
 +
2
 +
+y
 +
2
 +
+z
 +
2
 +
)
 +
2
 +
 +
1+y
 +
2
 +
+z
 +
2
 +
 +
 +
,
 +
and multiplying by
 +
x
 +
x (which is positive) gives:
 +
 
 +
x
 +
x
 +
4
 +
+
 +
y
 +
2
 +
+
 +
z
 +
2
 +
 +
x
 
(
 
(
t
+
1
3
 
 
+
 
+
3
+
y
t
+
2
)
+
+
t
+
z
4
+
2
=
+
)
3
+
(
t
+
x
3
+
2
 +
+
 +
y
 +
2
 
+
 
+
9
+
z
t
+
2
t
+
)
4
+
2
 
.
 
.
RHS≤
+
x
t
 
 
4
 
4
  /9
+
  +y
t+  
+
2
3
+
+z
t
+
2
3
 
 
   
 
   
 +
x
 
 
 +
 +
(x
 +
2
 +
+y
 +
2
 +
+z
 +
2
 +
)
 +
2
 
   
 
   
 +
x(1+y
 +
2
 +
+z
 +
2
 +
)
 
 
  =
+
  .
t
+
Similarly,
 +
 
 +
y
 +
y
 
4
 
4
+
+
9
+
z
+
2
(t+  
+
+
3
+
x
t
+
2
3
+
+
y
+
(
)=
+
1
t
+
+
 +
z
 +
2
 +
+
 +
x
 +
2
 +
)
 +
(
 +
x
 +
2
 +
+
 +
y
 +
2
 +
+
 +
z
 +
2
 +
)
 +
2
 +
,
 +
z
 +
z
 
4
 
4
+
+
9
+
x
+
2
+
+
3
+
y
3t+t
+
2
3
 
 
 
=
 
t
 
4
 
 
3(t
 
3
 
+3t)
 
 
=
 
t
 
4
 
 
3t
 
3
 
+9t
 
 
.
 
Wait, the original says:
 
 
 
 
 
t
+
z
 +
(
 +
1
 
+
 
+
t
+
x
3
+
2
3
 
t
 
4
 
=
 
3
 
t
 
3
 
 
+
 
+
9
+
y
t
+
2
 
27
 
t
 
4
 
 
1
 
 
(
 
t
 
 
3
 
 
)
 
)
 
(
 
(
t
+
x
3
+
2
+
+
9
+
y
 +
2
 +
+
 +
z
 +
2
 
)
 
)
+
2
0
+
.
+
y
t
 
 
4
 
4
 +
+z
 +
2
 +
+x
 +
2
 
   
 
   
t+
+
y
3
 
t
 
3
 
 
 
 
 +
 +
(x
 +
2
 +
+y
 +
2
 +
+z
 +
2
 +
)
 +
2
 
   
 
   
 +
y(1+z
 +
2
 +
+x
 +
2
 +
)
 
 
  =
+
  ,
t
+
z
 
4
 
4
 +
+x
 +
2
 +
+y
 +
2
 
   
 
   
3t
+
z
3
 
+9t−27
 
 
 
  ≤1⇔(t−3)(t
+
 
3
+
(x
  −9)≥0
+
2
But there is a discrepancy: The step from the previous expression to the next is not clear. Actually, the original says:
+
  +y
 
+
2
+
+z
t
+
2
+
+
  )
t
+
2
3
 
3
 
t
 
4
 
=
 
3
 
t
 
3
 
+
 
9
 
t
 
 
27
 
t
 
4
 
 
t
 
4
 
   
 
t+
 
3
 
t
 
3
 
 
   
 
   
 +
z(1+x
 +
2
 +
+y
 +
2
 +
)
 
 
   
+
  .
+
Summing these, we obtain:
=
+
 
t
+
 +
x
 +
x
 
4
 
4
+
+
3t
 
3
 
+9t−27
 
 
 
That seems to have an error: Actually, the original might have intended:
 
We have:
 
 
 
x
 
+
 
 
y
 
y
 +
2
 
+
 
+
 
z
 
z
+
+
2
 +
 +
1
 
(
 
(
 
x
 
x
y
+
2
 
+
 
+
 
y
 
y
z
+
2
 
+
 
+
 
z
 
z
x
+
2
 
)
 
)
(
+
2
 +
[
 +
 
x
 
x
 
+
 
+
y
+
+
 
z
 
)
 
 
3
 
 
x
 
x
y
 
z
 
 
(
 
(
x
 
2
 
+
 
 
y
 
y
 
2
 
2
Line 1,275: Line 1,432:
 
2
 
2
 
)
 
)
2
+
]
+
.
t
+
 +
x
 +
4
 +
+y
 +
2
 +
+z
 +
2
 +
 +
x
 +
 +
≤  
 +
(x
 +
2
 +
+y
 +
2
 +
+z
 +
2
 +
)
 +
2
 +
 +
1
 +
 +
[∑x+∑x(y
 +
2
 +
+z
 +
2
 +
)].
 +
Now, note that
 +
 
 +
 +
x
 +
(
 +
y
 +
2
 
+
 
+
(
+
z
t
 
 
2
 
2
/
 
3
 
 
)
 
)
t
 
t
 
4
 
/
 
9
 
 
=
 
=
t
+
 +
(
 +
x
 +
y
 +
2
 
+
 
+
t
+
x
3
+
z
/
+
2
3
+
)
t
 
4
 
/
 
9
 
 
=
 
=
9
+
(
t
+
x
 +
y
 +
2
 
+
 
+
t
+
x
3
+
z
/
+
2
3
+
)
t
 
4
 
=
 
9
 
t
 
 
+
 
+
3
+
(
t
+
y
3
+
z
t
+
2
4
 
=
 
3
 
t
 
3
 
 
+
 
+
9
+
y
t
+
x
t
 
4
 
.
 
(x  
 
 
2
 
2
+y
+
)
 +
+
 +
(
 +
z
 +
x
 
2
 
2
+z  
+
+
 +
z
 +
y
 
2
 
2
  )  
+
)
 +
=
 +
(
 +
x
 +
+
 +
y
 +
+
 +
z
 +
)
 +
(
 +
x
 +
y
 +
+
 +
y
 +
z
 +
+
 +
z
 +
x
 +
)
 +
 +
3
 +
x
 +
y
 +
z
 +
.
 +
∑x(y
 +
2
 +
+z
 +
2
 +
  )=∑(xy
 +
2
 +
+xz
 
2
 
2
   
+
  )=(xy  
x+y+z+(xy+yz+zx)(x+y+z)−3xyz
 
 
 
t
 
4
 
/9
 
t+(t
 
 
2
 
2
  /3)t
+
  +xz
+
2
  =  
+
)+(yz
t
+
2
 +
+yx
 +
2
 +
)+(zx
 +
2
 +
  +zy
 +
2
 +
)=(x+y+z)(xy+yz+zx)−3xyz.
 +
Thus,
 +
 
 +
 +
x
 +
x
 
4
 
4
/9
+
+
t+t  
+
y
 +
2
 +
+
 +
z
 +
2
 +
 +
t
 +
+
 +
t
 +
(
 +
x
 +
y
 +
+
 +
y
 +
z
 +
+
 +
z
 +
x
 +
)
 +
 
3
 
3
/3
+
x
+
y
=9
+
z
t
+
(
 +
x
 +
2
 +
+
 +
y
 +
2
 +
+
 +
z
 +
2
 +
)
 +
2
 +
.
 +
 +
x
 
4
 
4
 +
+y
 +
2
 +
+z
 +
2
 
   
 
   
t+t
+
x
3
 
/3
 
 
 
  =
+
 
t
+
(x
4
+
2
   
+
  +y
9t+3t
+
2
3
+
+z
 +
2
 +
)
 +
2
 
   
 
   
 +
t+t(xy+yz+zx)−3xyz
 
 
  =
+
  .
t
+
Since
4
+
x
+
y
3t
+
z
 +
 +
1
 +
xyz≥1, we have
 +
 
3
 
3
+9t
+
x
+
y
.
+
z
But then the original writes:
 
 
 
 
 
t
+
 +
3
 +
−3xyz≤−3. Also,
 +
x
 +
y
 +
+
 +
y
 +
z
 
+
 
+
 +
z
 +
x
 +
 
t
 
t
 +
2
 
3
 
3
 +
xy+yz+zx≤
 
3
 
3
 +
t
 +
2
 +
 +
 +
  and
 +
x
 +
2
 +
+
 +
y
 +
2
 +
+
 +
z
 +
2
 +
 
t
 
t
4
+
2
=
 
3
 
t
 
 
3
 
3
+
+
x
9
+
2
t
+
+y
+
2
27
+
+z
t
+
2
4
+
 
 
t
 
4
 
   
 
t+
 
 
3
 
3
 
t  
 
t  
3
+
2
 
   
 
   
 
 
   
+
  , so
+
 
=
+
(
t
+
x
4
+
2
+
+
3t
+
y
3
+
2
+9t−27
 
 
 
That is inconsistent. Possibly there is a misprint. Actually, the original then says:
 
 
 
3
 
t
 
3
 
 
+
 
+
9
+
z
t
+
2
+
)
27
+
2
t
+
4
 
 
1
 
 
 
(
 
(
 
t
 
t
+
2
 
3
 
3
 
)
 
)
(
+
2
 +
=
 
t
 
t
3
+
4
+
9
 +
.
 +
(x
 +
2
 +
+y
 +
2
 +
+z
 +
2
 +
)
 +
2
 +
≥(
 +
3
 +
t
 +
2
 +
 +
 +
)
 +
2
 +
=
 
9
 
9
)
 
 
0.
 
 
t  
 
t  
 
4
 
4
 
   
 
   
3t
 
3
 
+9t−27
 
 
 
  ≤1⇔(t−3)(t
+
  .
3
+
Therefore,
−9)≥0.
 
But if we had <math>\frac{3t^3+9t}{t^4}</math>, then we would need to show that <math>\frac{3t^3+9t}{t^4} \leq 1</math> for <math>t \geq 3</math>, which is <math>3t^3+9t \leq t^4</math> or <math>t^4 - 3t^3 - 9t \geq 0</math>, i.e., <math>t(t^3-3t^2-9) \geq 0</math>. That is not the same as <math>(t-3)(t^3-9) \geq 0</math>. So there is a discrepancy.
 
  
Let's re-read the original:
+
"RHS <math>\leq \frac{x+y+z+(xy+yz+zx)(x+y+z)-3xyz}{(x^2 + y^2 + z^2)^2}
+
x
\leq \frac{t + \frac{t^3}{3}}{t^4} = \frac{3t^3 + 9t - 27}{t^4} \leq 1</math>"
+
x
 
+
4
Wait, the original has:
 
 
 
 
t
 
 
+
 
+
t
+
y
 +
2
 +
+
 +
z
 +
2
 +
 +
t
 +
+
 +
t
 +
 +
t
 +
2
 
3
 
3
 +
 
3
 
3
 +
(
 
t
 
t
4
+
2
 +
/
 +
3
 +
)
 +
2
 
=
 
=
3
 
 
t
 
t
3
 
 
+
 
+
9
 
 
t
 
t
 +
3
 +
3
 
 
27
+
3
 
t
 
t
 
4
 
4
+
/
t
+
9
4
+
=
+
9
t+
+
(
3
+
t
t
 
3
 
 
 
 
 
=
 
t
 
4
 
 
3t
 
3
 
+9t−27
 
 
 
But <math>\frac{t + \frac{t^3}{3}}{t^4} = \frac{3t + t^3}{3t^4} = \frac{t^3+3t}{3t^4}</math>, not <math>\frac{3t^3+9t-27}{t^4}</math>. So there is a mistake.
 
 
 
Maybe it is:
 
 
 
 
t
 
 
+
 
+
 
t
 
t
Line 1,522: Line 1,782:
 
 
 
3
 
3
(
 
x
 
2
 
+
 
y
 
2
 
+
 
z
 
2
 
 
)
 
)
2
+
t
+
4
 +
=
 +
9
 
t
 
t
 
+
 
+
 +
3
 
t
 
t
3
 
 
3
 
3
 
 
3
+
27
t
 
4
 
/
 
9
 
=
 
9
 
(
 
t
 
+
 
t
 
3
 
/
 
3
 
 
3
 
)
 
t
 
4
 
=
 
9
 
t
 
+
 
3
 
t
 
3
 
 
27
 
 
t
 
t
 
4
 
4
Line 1,581: Line 1,807:
 
t
 
t
 
4
 
4
+
.
(x  
+
2
+
x  
 +
4
 
  +y  
 
  +y  
 
2
 
2
 
  +z  
 
  +z  
2
 
)
 
 
2
 
2
 
   
 
   
t+  
+
x
 +
 +
 +
(t
 +
2
 +
/3)
 +
2
 +
 +
t+t⋅
 
3
 
3
 
t  
 
t  
3
+
2
 
   
 
   
 
 
 
  −3
 
  −3
 
 
 
+
  =
 
t  
 
t  
 
4
 
4
Line 1,615: Line 1,848:
 
4
 
4
 
   
 
   
9(t+t
+
9(t+  
 
3
 
3
  /3−3)
+
t
 +
3
 +
   
 +
 +
−3)
 
 
 
  =  
 
  =  
Line 1,635: Line 1,872:
 
  +9t−27
 
  +9t−27
 
 
   
+
  .
because we also subtract 3xyz? Actually, note that the numerator is:
+
We now show that
x+y+z + (xy+yz+zx)(x+y+z) - 3xyz.
 
And we have xyz >= 1, so -3xyz <= -3. So we can bound:
 
x+y+z + (xy+yz+zx)(x+y+z) - 3xyz <= t + (t^2/3)*t - 3 = t + t^3/3 - 3.
 
And then denominator: (x^2+y^2+z^2)^2 >= (t^2/3)^2 = t^4/9.
 
So indeed:
 
RHS <= (t + t^3/3 - 3) / (t^4/9) = 9(t + t^3/3 - 3)/t^4 = (9t + 3t^3 - 27)/t^4 = (3t^3+9t-27)/t^4.
 
So that step is correct:
 
  
RHS
 
 
 
3
 
3
 
t
 
t
Line 1,657: Line 1,885:
 
t
 
t
 
4
 
4
.
+
RHS≤
+
1.
 
t  
 
t  
 
4
 
4
Line 1,666: Line 1,894:
 
  +9t−27
 
  +9t−27
 
 
  .
+
  ≤1.
Then we want to show that this is ≤ 1, i.e.,
+
This is equivalent to
  
3
 
t
 
3
 
+
 
9
 
t
 
 
27
 
t
 
4
 
 
1
 
 
 
3
 
3
 
t
 
t
Line 1,707: Line 1,922:
 
 
 
0.
 
0.
t
+
3t  
4
 
 
3t  
 
3
 
+9t−27
 
 
≤1⇔3t
 
 
3
 
3
 
  +9t−27≤t  
 
  +9t−27≤t  
Line 1,723: Line 1,931:
 
3
 
3
 
  −9t+27≥0.
 
  −9t+27≥0.
Factor: t^4 - 3t^3 - 9t + 27 = (t-3)(t^3 - 9) ? Let's check: (t-3)(t^3 - 9) = t^4 - 9t - 3t^3 + 27 = t^4 - 3t^3 - 9t + 27. Yes.
+
Factor the left-hand side:
So we need (t-3)(t^3-9) >= 0. For t>=3, t-3>=0 and t^3-9>=0, so indeed it holds. So RHS ≤ 1.
 
  
Then the original says: "By AM-GM: LHS ≥ ...". But careful: The LHS here is the left-hand side of the inequality we want to prove? Actually, we want to prove that LHS (which is the sum of fractions with x^4) is at least 1. So we need to show LHS ≥ 1.
+
t
 
+
4
The original then says:
 
LHS ≥ (x^3+y^3+z^3)^2 / (x^6+y^6+z^6+2(x^2y^2+y^2z^2+z^2x^2)) ≥ 1.
 
That is by Cauchy-Schwarz? Actually, by Titu's lemma (Cauchy-Schwarz in Engel form):
 
∑ x^4/(x^4+y^2+z^2) ≥ (∑ x^2)^2 / ∑ (x^4+y^2+z^2) = (x^2+y^2+z^2)^2 / (∑ x^4 + 2∑ x^2) but careful: Actually, the denominators are different: The first term denominator is x^4+y^2+z^2, so if we sum over cyclic, then the denominator sum becomes (x^4+y^4+z^4) + (y^2+z^2) + (z^2+x^2) + (x^2+y^2) = x^4+y^4+z^4 + 2(x^2+y^2+z^2). So then by Cauchy, we have:
 
LHS = ∑ x^4/(x^4+y^2+z^2) ≥ (x^2+y^2+z^2)^2 / (x^4+y^4+z^4+2(x^2+y^2+z^2)).
 
But the original writes:
 
LHS ≥ (x^3+y^3+z^3)^2 / (x^6+y^6+z^6+2(x^2y^2+y^2z^2+z^2x^2)).
 
That is different: They have x^3 in numerator squared, and denominator: x^6+y^6+z^6+2(x^2y^2+...). Actually, note that (x^3)^2 = x^6. So it is similar but with squares? Actually, if we set a = x^2, then a^2 = x^4. So it's not exactly.
 
 
 
Wait, maybe they used:
 
By Cauchy, (∑ x^4/(x^4+y^2+z^2)) (∑ x^4(x^4+y^2+z^2)) ≥ (∑ x^4)^2.
 
But then ∑ x^4(x^4+y^2+z^2) = ∑ (x^8 + x^4y^2 + x^4z^2) = ∑ x^8 + ∑ x^4y^2 + ∑ x^4z^2.
 
That is not the same as x^6+y^6+z^6+2(x^2y^2+...)? Actually, x^6? No.
 
 
 
Alternatively, they might have used the inequality:
 
∑ x^4/(x^4+y^2+z^2) ≥ (∑ x^2)^2 / (∑ x^4 + 2∑ x^2)
 
but then they claim that is ≥ 1 if (x^2+y^2+z^2)^2 ≥ x^4+y^4+z^4+2(x^2+y^2+z^2) which is not necessarily true.
 
 
 
Maybe they used a different approach: They said:
 
LHS ≥ (x^3+y^3+z^3)^2 / (x^6+y^6+z^6+2(x^2y^2+y^2z^2+z^2x^2)).
 
That would come from Cauchy if we write:
 
(∑ x^4/(x^4+y^2+z^2)) (∑ (x^4+y^2+z^2)) ≥ (∑ x^2)^2? Not exactly.
 
 
 
Wait, consider:
 
We want to lower bound LHS = ∑ x^4/(x^4+y^2+z^2).
 
By Cauchy, (∑ x^4/(x^4+y^2+z^2)) (∑ x^4(x^4+y^2+z^2)) ≥ (∑ x^4)^2.
 
But then that gives LHS ≥ (∑ x^4)^2 / (∑ (x^8 + x^4y^2+x^4z^2)) = (∑ x^4)^2 / (∑ x^8 + ∑ x^4y^2+∑ x^4z^2).
 
And note that ∑ x^8 = x^8+y^8+z^8, and ∑ x^4y^2+∑ x^4z^2 = ∑ x^4(y^2+z^2) = ∑ x^4((x^2+y^2+z^2)-x^2) = (x^2+y^2+z^2)∑ x^4 - ∑ x^6.
 
So then denominator = ∑ x^8 + (x^2+y^2+z^2)∑ x^4 - ∑ x^6.
 
That is not the same as x^6+y^6+z^6+2(x^2y^2+y^2z^2+z^2x^2) unless further conditions.
 
 
 
Alternatively, maybe they used:
 
By Titu's lemma:
 
∑ x^4/(x^4+y^2+z^2) ≥ (∑ x^2)^2 / (∑ (x^4+y^2+z^2)) = (x^2+y^2+z^2)^2 / (∑ x^4 + 2∑ x^2).
 
But then they claim that is ≥ 1 if (x^2+y^2+z^2)^2 ≥ ∑ x^4+2∑ x^2, i.e., 2(x^2y^2+y^2z^2+z^2x^2) ≥ 2(x^2+y^2+z^2) or x^2y^2+y^2z^2+z^2x^2 ≥ x^2+y^2+z^2, which is not necessarily true.
 
 
 
Given that the original text says:
 
"LHS ≥ \frac{(x^3 + y^3 + z^3)^2}{x^6 + y^6 + z^6 + 2(x^2y^2 + y^2z^2 + z^2x^2)} \geq 1"
 
Maybe it is actually:
 
LHS = ∑ x^4/(x^4+y^2+z^2) and they use the substitution: Let a = x^2, b = y^2, c = z^2. Then LHS = ∑ a^2/(a^2+b+c). And then by Cauchy,
 
(∑ a^2/(a^2+b+c)) (∑ a^2(a^2+b+c)) ≥ (∑ a^2)^2.
 
But ∑ a^2(a^2+b+c) = ∑ (a^4 + a^2b + a^2c) = ∑ a^4 + ∑ a^2(b+c) = ∑ a^4 + ∑ a^2((a+b+c)-a) = ∑ a^4 + (a+b+c)∑ a^2 - ∑ a^3.
 
So then LHS ≥ (∑ a^2)^2 / (∑ a^4 + (a+b+c)∑ a^2 - ∑ a^3). That doesn't simplify to the given.
 
 
 
Alternatively, they might have used the inequality:
 
a^2/(a^2+b+c) ≥ (a^3)^2/(a^6 + a^3(b+c))? That is not standard.
 
 
 
Wait, the given expression:
 
\frac{(x^3+y^3+z^3)^2}{x^6+y^6+z^6+2(x^2y^2+y^2z^2+z^2x^2)}
 
Notice that x^6+y^6+z^6+2(x^2y^2+y^2z^2+z^2x^2) = (x^2+y^2+z^2)^3? Actually, (x^2+y^2+z^2)^3 = x^6+y^6+z^6+3(x^2y^2(x^2+y^2)+... so no.
 
 
 
Maybe it's a misprint: They might have meant:
 
LHS ≥ \frac{(x^2+y^2+z^2)^2}{x^4+y^4+z^4+2(x^2+y^2+z^2)}.
 
But then they claim that is ≥ 1 if (x^2+y^2+z^2)^2 ≥ x^4+y^4+z^4+2(x^2+y^2+z^2) which is equivalent to 2(x^2y^2+y^2z^2+z^2x^2) ≥ 2(x^2+y^2+z^2) or x^2y^2+y^2z^2+z^2x^2 ≥ x^2+y^2+z^2. That is not generally true.
 
 
 
Given the subsequent steps:
 
They then say:
 
≥ 1 ⇔ x^3y^3+y^3z^3+z^3x^3 ≥ x^2y^2+y^2z^2+z^2x^2 ⇔ a^3+b^3+c^3 ≥ a^2+b^2+c^2, where a=xy, b=yz, c=zx.
 
So indeed, they are comparing:
 
(x^3+y^3+z^3)^2 and x^6+y^6+z^6+2(x^2y^2+y^2z^2+z^2x^2).
 
Note that (x^3+y^3+z^3)^2 = x^6+y^6+z^6+2(x^3y^3+x^3z^3+y^3z^3).
 
So the inequality
 
\frac{(x^3+y^3+z^3)^2}{x^6+y^6+z^6+2(x^2y^2+y^2z^2+z^2x^2)} \geq 1
 
is equivalent to
 
x^6+y^6+z^6+2(x^3y^3+x^3z^3+y^3z^3) \geq x^6+y^6+z^6+2(x^2y^2+y^2z^2+z^2x^2)
 
i.e., x^3y^3+x^3z^3+y^3z^3 \geq x^2y^2+y^2z^2+z^2x^2.
 
So that step is correct: They claim that LHS (the sum of fractions) is at least that fraction, and then that fraction is at least 1 if x^3y^3+... >= x^2y^2+... So then they set a=xy, b=yz, c=zx, so that a^3+b^3+c^3 >= a^2+b^2+c^2.
 
And then they note that abc = x^2y^2z^2, and since x,y,z are positive with xyz>=1, then abc>=1.
 
And then they use Holder: (a^2+b^2+c^2)^3 <= 3(a^3+b^3+c^3)^2 <= (a^3+b^3+c^3)^3? Actually, they write:
 
By Holder: (a^2+b^2+c^2)^3 <= 3(a^3+b^3+c^3)^2 <= (a^3+b^3+c^3)^3 ⇒ a^2+b^2+c^2 <= a^3+b^3+c^3.
 
Wait, check: Holder says: (a^3+b^3+c^3)(1+1+1)(1+1+1) >= (a+b+c)^3, but that's not directly.
 
They claim: (a^2+b^2+c^2)^3 <= 3(a^3+b^3+c^3)^2. Is that true?
 
By power mean, ( (a^3+b^3+c^3)/3 )^(1/3) >= ( (a^2+b^2+c^2)/3 )^(1/2) so (a^3+b^3+c^3) >= 3^{-1/2} (a^2+b^2+c^2)^(3/2)? That gives (a^3+b^3+c^3)^2 >= (1/3)(a^2+b^2+c^2)^3, so actually (a^2+b^2+c^2)^3 <= 3(a^3+b^3+c^3)^2. So that is true.
 
Then they claim: 3(a^3+b^3+c^3)^2 <= (a^3+b^3+c^3)^3, which would require a^3+b^3+c^3 >= 3. And since a,b,c are such that abc>=1, by AM-GM, a^3+b^3+c^3 >= 3(abc) >= 3. So indeed, a^3+b^3+c^3 >= 3. Then we have:
 
(a^2+b^2+c^2)^3 <= 3(a^3+b^3+c^3)^2 <= (a^3+b^3+c^3)^3, so taking cube roots: a^2+b^2+c^2 <= a^3+b^3+c^3.
 
So that step is valid.
 
 
 
But then the claim that LHS >= that fraction is not justified in the text. They simply say "By AM-GM:" and then write that inequality. Possibly they meant by Cauchy-Schwarz? Let's check: We want to show:
 
∑ x^4/(x^4+y^2+z^2) >= (x^3+y^3+z^3)^2 / (x^6+y^6+z^6+2(x^2y^2+y^2z^2+z^2x^2)).
 
This is equivalent to:
 
(∑ x^4/(x^4+y^2+z^2)) (x^6+y^6+z^6+2(x^2y^2+y^2z^2+z^2x^2)) >= (x^3+y^3+z^3)^2.
 
Now, by Cauchy, we have:
 
(∑ x^4/(x^4+y^2+z^2)) (∑ x^4(x^4+y^2+z^2)) >= (∑ x^4)^2.
 
But we need (∑ x^3)^2 on the right. So if we can show that (∑ x^4)^2 >= (∑ x^3)^2, i.e., ∑ x^4 >= ∑ x^3, that is not always true. Alternatively, if we use the weights differently:
 
We want to relate x^4(x^4+y^2+z^2) to something like x^6? Not sure.
 
 
 
Maybe they used the following:
 
x^4/(x^4+y^2+z^2) >= x^6/(x^6+x^2y^2+x^2z^2) because multiply numerator and denominator by x^2 gives: x^6/(x^6+x^2y^2+x^2z^2). And then sum cyclically:
 
LHS >= ∑ x^6/(x^6+x^2y^2+x^2z^2).
 
Then by Cauchy,
 
∑ x^6/(x^6+x^2y^2+x^2z^2) >= (∑ x^3)^2 / (∑ x^6+∑ x^2y^2+∑ x^2z^2) = (∑ x^3)^2 / (∑ x^6+2∑ x^2y^2).
 
That is exactly the claimed lower bound. And the inequality
 
x^4/(x^4+y^2+z^2) >= x^6/(x^6+x^2y^2+x^2z^2)
 
is equivalent to cross multiply: x^4(x^6+x^2y^2+x^2z^2) >= x^6(x^4+y^2+z^2) => x^10+x^6y^2+x^6z^2 >= x^10+x^6y^2+x^6z^2, which is equality! Actually, check:
 
x^4/(x^4+y^2+z^2) >= x^6/(x^6+x^2y^2+x^2z^2)
 
Multiply both sides by positive denominators:
 
x^4(x^6+x^2y^2+x^2z^2) >= x^6(x^4+y^2+z^2)
 
=> x^10 + x^6y^2 + x^6z^2 >= x^10 + x^6y^2 + x^6z^2.
 
So it's actually equality. So indeed,
 
x^4/(x^4+y^2+z^2) = x^6/(x^6+x^2y^2+x^2z^2).
 
Because multiply numerator and denominator by x^2 gives that provided x>0. So we have:
 
LHS = ∑ x^4/(x^4+y^2+z^2) = ∑ x^6/(x^6+x^2y^2+x^2z^2).
 
Then by Cauchy-Schwarz,
 
∑ x^6/(x^6+x^2y^2+x^2z^2) ≥ (∑ x^3)^2 / (∑ x^6 + ∑ x^2y^2+∑ x^2z^2) = (∑ x^3)^2 / (∑ x^6+2∑ x^2y^2).
 
So that step is justified.
 
 
 
So we will include that step.
 
 
 
Now, let's write the LaTeX code accordingly.
 
 
 
I'll structure it as:
 
 
 
\documentclass{article}
 
\usepackage{amsmath, amssymb}
 
\begin{document}
 
 
 
\section*{Solution}
 
 
 
First, we can easily see that
 
 
 
x
 
5
 
 
 
x
+
3
2
+
t
x
+
3
5
+
 +
9
 +
t
 
+
 
+
y
+
27
2
+
=
+
+
(
z
+
t
2
+
+
3
x
+
)
4
+
(
 +
t
 +
3
 
 
x
+
9
x
+
)
 +
.
 +
t
 
4
 
4
+
+
−3t
y
+
3
 +
−9t+27=(t−3)(t
 +
3
 +
−9).
 +
For
 +
t
 +
 +
3
 +
t≥3, we have
 +
t
 +
 +
3
 +
 +
0
 +
t−3≥0 and
 +
t
 +
3
 +
 +
9
 +
 +
18
 +
t
 +
3
 +
−9≥18, so indeed
 +
(
 +
t
 +
 +
3
 +
)
 +
(
 +
t
 +
3
 +
 +
9
 +
)
 +
 +
0
 +
(t−3)(t
 +
3
 +
−9)≥0. Hence,
 +
 
 +
 +
x
 +
x
 +
4
 +
+
 +
y
 
2
 
2
 
+
 
+
 
z
 
z
 
2
 
2
x
+
5
+
1.
+y
+
(2)
2
+
+z
 
2
 
 
x
 
5
 
−x
 
2
 
 
 
 
 
x  
 
x  
 
4
 
4
Line 1,895: Line 2,022:
 
2
 
2
 
   
 
   
x  
+
x
4
 
−x
 
 
 
   
+
  ≤1.(2)
since
+
Now, we estimate the left-hand side of (1). Note that
  
 
x
 
x
5
+
4
 
 
x
 
x
2
+
4
x
 
5
 
 
+
 
+
 
y
 
y
Line 1,915: Line 2,037:
 
z
 
z
 
2
 
2
+
=
 
x
 
x
4
+
6
 
 
x
 
x
 +
6
 +
+
 
x
 
x
4
+
2
+
 
 
y
 
y
 
2
 
2
 
+
 
+
z
 
2
 
=
 
x
 
(
 
 
x
 
x
 
1
 
)
 
 
2
 
2
(
 
x
 
2
 
+
 
x
 
+
 
1
 
)
 
(
 
y
 
2
 
+
 
 
z
 
z
 
2
 
2
)
+
,
(
+
x  
x
+
4
5
+
+y  
+
 
y
 
 
2
 
2
+
+
+z  
z
 
 
2
 
2
)
+
(
+
x
x
+
4
4
+
+
+
y
+
=
 +
x  
 +
6
 +
+x
 
2
 
2
+
+
y
z
 
 
2
 
2
)
+
  +x
 
0
 
,
 
x
 
5
 
  +y
 
 
2
 
2
  +z  
+
  z  
 
2
 
2
 
   
 
   
 
x  
 
x  
5
+
6
−x
 
2
 
 
   
 
   
 
 
 
+
  ,
x  
+
and similarly for the other terms. So,
 +
 
 +
 +
x
 +
4
 +
x
 
4
 
4
+y  
+
+
 +
y
 +
2
 +
+
 +
z
 
2
 
2
+z  
+
=
 +
 +
x
 +
6
 +
x
 +
6
 +
+
 +
x
 +
2
 +
y
 +
2
 +
+
 +
x
 +
2
 +
z
 
2
 
2
+
.
 +
 
x  
 
x  
 
4
 
4
−x
 
 
=
 
(x
 
5
 
 
  +y  
 
  +y  
 
2
 
2
 
  +z  
 
  +z  
 
2
 
2
  )(x  
+
   
 +
x  
 
4
 
4
  +y
+
   
2
+
  +z
+
  =∑
2
+
x
  )
+
6
x(x−1)
+
  +x  
 
2
 
2
  (x
+
  y
 
2
 
2
  +x+1)(y
+
  +x  
 
2
 
2
  +z  
+
  z  
 
2
 
2
  )
+
   
 +
x
 +
6
 +
 
 
  ≥0,
+
  .
which holds for all positive real numbers <math>x, y, z</math>.
+
By the Cauchy–Schwarz inequality (Titu's lemma), we have:
 
 
Thus, it is enough to prove that
 
  
 +
 
x
 
x
4
+
6
 
 
x
 
x
 +
6
 +
+
 
x
 
x
4
+
2
+
 
 
y
 
y
 
2
 
2
 
+
 
+
 +
x
 +
2
 
z
 
z
 
2
 
2
 +
 +
(
 +
x
 +
3
 
+
 
+
 
y
 
y
4
+
3
 
y
 
y
 
4
 
 
+
 
+
 
z
 
z
 +
3
 +
)
 
2
 
2
 +
 +
(
 +
x
 +
6
 
+
 
+
 
x
 
x
 +
2
 +
y
 
2
 
2
 
+
 
+
 +
x
 +
2
 
z
 
z
4
+
2
+
)
 +
=
 +
(
 +
x
 +
3
 +
+
 +
y
 +
3
 +
+
 
z
 
z
z
+
3
4
+
)
+
+
2
 
x
 
x
2
+
6
 
+
 
+
 
y
 
y
 +
6
 +
+
 +
z
 +
6
 +
+
 +
2
 +
(
 +
x
 +
2
 +
y
 +
2
 +
+
 +
y
 +
2
 +
z
 +
2
 +
+
 +
z
 
2
 
2
+
x
0.
 
x  
 
4
 
+y
 
2
 
+z
 
 
2
 
2
+
)
 +
.
 +
(3)
 +
 
x  
 
x  
4
+
6
  −x
+
  +x
+
2
  +
+
  y  
y  
 
4
 
+z
 
 
2
 
2
 
  +x  
 
  +x  
 
2
 
2
 +
z
 +
2
 +
 +
x
 +
6
 
   
 
   
y
 
4
 
−y
 
 
 
  +  
+
z
+
∑(x
4
+
6
 +
  +x
 +
2
 +
y
 +
2
 
  +x  
 
  +x  
 
2
 
2
 +
z
 +
2
 +
)
 +
(x
 +
3
 
  +y  
 
  +y  
 +
3
 +
+z
 +
3
 +
)
 
2
 
2
 
   
 
   
z
 
4
 
−z
 
 
 
  ≥0.
+
  =
Rewriting, we need to show
+
x
 
+
6
x
+
+y
4
+
6
x
+
+z
4
+
6
+
+
+2(x
y
 
 
2
 
2
+
+
y
z
 
 
2
 
2
+
+
+y  
y
 
4
 
y
 
4
 
+
 
z
 
 
2
 
2
+
+
z
x
 
 
2
 
2
+
+
+z  
z
 
4
 
z
 
4
 
+
 
x
 
 
2
 
2
+
+
x
y
 
 
2
 
2
+
)
 +
(x
 +
3
 +
+y
 +
3
 +
+z
 +
3
 +
)
 +
2
 +
 +
 +
.(3)
 +
We claim that
 +
 
 +
(
 
x
 
x
x
+
3
4
 
 
+
 
+
 
y
 
y
2
+
3
 
+
 
+
 
z
 
z
 +
3
 +
)
 
2
 
2
 +
 +
x
 +
6
 
+
 
+
 
y
 
y
y
+
6
4
 
 
+
 
+
 
z
 
z
 +
6
 +
+
 
2
 
2
+
+
(
 
x
 
x
 
2
 
2
+
+
y
z
 
z
 
4
 
+
 
x
 
 
2
 
2
 
+
 
+
 
y
 
y
 
2
 
2
 +
z
 +
2
 +
+
 +
z
 +
2
 +
x
 +
2
 +
)
 
.
 
.
x  
+
(x  
4
+
3
 
  +y  
 
  +y  
 +
3
 +
+z
 +
3
 +
)
 
2
 
2
 +
≥x
 +
6
 +
+y
 +
6
 
  +z  
 
  +z  
 +
6
 +
+2(x
 +
2
 +
y
 +
2
 +
+y
 +
2
 +
z
 
2
 
2
 
x
 
4
 
 
 
+
 
y
 
4
 
 
  +z  
 
  +z  
 
2
 
2
  +x  
+
  x  
 
2
 
2
   
+
  ).
y
+
Expanding the left-hand side:
4
+
 
+
(
+
x
+  
+
3
z  
+
+
4
+
y
+x
+
3
 +
+
 +
z
 +
3
 +
)
 
2
 
2
+y  
+
=
 +
x
 +
6
 +
+
 +
y
 +
6
 +
+
 +
z
 +
6
 +
+
 
2
 
2
+
(
z  
+
x
4
+
3
+
y
+
3
+
+
x  
+
y
4
+
3
 +
z
 +
3
 +
+
 +
z
 +
3
 +
x
 +
3
 +
)
 +
.
 +
(x  
 +
3
 
  +y  
 
  +y  
2
+
3
 
  +z  
 
  +z  
 +
3
 +
)
 
2
 
2
   
+
  =x  
x
+
6
+
  +y  
  +  
+
6
y  
 
4
 
 
  +z  
 
  +z  
2
+
6
  +x  
+
  +2(x  
2
+
3
   
+
  y  
y
+
3
 
+
 
z
 
4
 
+x
 
2
 
 
  +y  
 
  +y  
2
+
3
   
+
  z
z
+
3
+
+z  
  .
+
3
Let <math>t = x + y + z</math>. By the AM–GM inequality, we have <math>t \geq 3\sqrt[3]{xyz} \geq 3</math>, and also <math>xy + yz + zx \leq \frac{t^2}{3} \leq x^2 + y^2 + z^2</math>.
+
x
 +
3
 +
  ).
 +
Thus, the inequality is equivalent to
  
Now, estimate the right-hand side (RHS). By the Cauchy–Schwarz inequality,
 
 
(
 
 
x
 
x
2
+
3
 +
y
 +
3
 
+
 
+
 
y
 
y
2
+
3
 +
z
 +
3
 
+
 
+
 
z
 
z
 +
3
 +
x
 +
3
 +
 +
x
 
2
 
2
)
+
y
 
2
 
2
 
(
 
x
 
4
 
 
+
 
+
 
y
 
y
 
2
 
2
+
 
 
z
 
z
2
 
)
 
(
 
1
 
+
 
y
 
 
2
 
2
 
+
 
+
 
z
 
z
 
2
 
2
)
+
x
,
 
(x  
 
 
2
 
2
 +
.
 +
(4)
 +
x
 +
3
 +
y
 +
3
 
  +y  
 
  +y  
2
+
3
 +
z
 +
3
 
  +z  
 
  +z  
 +
3
 +
x
 +
3
 +
≥x
 
2
 
2
  )
+
  y
 
2
 
2
≤(x
 
4
 
 
  +y  
 
  +y  
 +
2
 +
z
 
2
 
2
 
  +z  
 
  +z  
 
2
 
2
  )(1+y
+
  x
 
2
 
2
  +z  
+
  .(4)
2
+
Let
),
+
a
so
+
=
 
+
x
 +
y
 +
a=xy,
 +
b
 +
=
 +
y
 +
z
 +
b=yz,
 +
c
 +
=
 +
z
 
x
 
x
 +
c=zx. Then
 +
a
 +
b
 +
c
 +
=
 
x
 
x
4
+
2
+
 
 
y
 
y
 
2
 
2
+
 
 
z
 
z
 
2
 
2
+
x
 
(
 
 
1
 
1
 +
abc=x
 +
2
 +
y
 +
2
 +
z
 +
2
 +
≥1, and inequality (4) becomes:
 +
 +
a
 +
3
 
+
 
+
y
+
b
2
+
3
 
+
 
+
z
+
c
2
+
3
)
+
(
+
a
x
 
 
2
 
2
 
+
 
+
y
+
b
 
2
 
2
 
+
 
+
z
+
c
2
 
)
 
 
2
 
2
 
.
 
.
x
+
a
4
+
3
  +y
+
  +b
 +
3
 +
+c
 +
3
 +
≥a
 
2
 
2
  +z
+
  +b
 
2
 
2
   
+
  +c
x
 
 
 
(x
 
 
2
 
2
+y
 
2
 
+z
 
2
 
)
 
2
 
 
x(1+y
 
2
 
+z
 
2
 
)
 
 
 
  .
 
  .
Cyclically summing, we get
+
We now prove this. By the AM–GM inequality,  
\begin{align*}
+
a
\text{RHS} &\leq \frac{x(1+y^2+z^2) + y(1+z^2+x^2) + z(1+x^2+y^2)}{(x^2+y^2+z^2)^2} \
 
&= \frac{x+y+z + (xy+yz+zx)(x+y+z) - 3xyz}{(x^2+y^2+z^2)^2}.
 
\end{align*}
 
Since <math>xyz \geq 1</math>, we have <math>-3xyz \leq -3</math>. Also, <math>xy+yz+zx \leq \frac{t^2}{3}</math> and <math>x^2+y^2+z^2 \geq \frac{t^2}{3}</math>. Hence,
 
\begin{align*}
 
\text{RHS} &\leq \frac{t + \frac{t^2}{3} \cdot t - 3}{(t^2/3)^2} = \frac{t + \frac{t^3}{3} - 3}{t^4/9} = \frac{9(t + t^3/3 - 3)}{t^4} = \frac{9t + 3t^3 - 27}{t^4} = \frac{3t^3 + 9t - 27}{t^4}.
 
\end{align*}
 
We claim that
 
 
 
 
3
 
3
t
+
+
 +
b
 
3
 
3
 
+
 
+
9
+
c
t
+
3
+
27
+
3
t
+
a
4
+
b
+
c
1.
+
t
+
3
4
+
a
   
+
3
3t
+
  +b
 +
3
 +
+c
 
3
 
3
  +9t−27
+
  ≥3abc≥3. Also, by the power mean inequality, we have:
 
≤1.
 
This is equivalent to
 
  
 +
(
 +
a
 
3
 
3
t
+
+
 +
b
 
3
 
3
 
+
 
+
9
+
c
t
+
3
+
3
27
+
)
+
1
t
+
/
4
 
 
t
 
4
 
 
 
3
 
3
t
+
3
+
(
+
a
9
+
2
t
+
+
 +
b
 +
2
 
+
 
+
27
+
c
+
2
0.
 
3t
 
 
3
 
3
+9t−27≤t
+
)
4
+
1
⇔t
+
/
4
+
2
−3t
+
,
 +
(
 
3
 
3
−9t+27≥0.
+
a
Factoring, we have
 
 
 
t
 
4
 
 
 
3
 
3
t
+
+b
 
3
 
3
+
+c
9
 
t
 
+
 
27
 
=
 
(
 
t
 
 
 
3
 
3
)
+
(
+
t
+
)  
 +
1/3
 +
(  
 
3
 
3
+
a
9
+
2
)
+
+b
+
2
0
+
+c
,
+
2
t
+
4
+
−3t
+
)  
 +
1/2
 +
,
 +
which implies
 +
 
 +
a
 
3
 
3
−9t+27=(t−3)(t
+
+
 +
b
 
3
 
3
−9)≥0,
 
which holds for <math>t \geq 3</math>. Therefore, RHS <math>\leq 1</math>.
 
 
Now, estimate the left-hand side (LHS). Note that for <math>x > 0</math>, we have
 
 
x
 
4
 
x
 
4
 
 
+
 
+
y
+
c
 +
3
 +
 +
3
 +
(
 +
a
 
2
 
2
 
+
 
+
z
+
b
 
2
 
2
=
 
x
 
6
 
x
 
6
 
 
+
 
+
x
+
c
 
2
 
2
y
+
3
 +
)
 +
3
 +
/
 +
2
 +
=
 +
(
 +
a
 +
2
 +
+
 +
b
 
2
 
2
 
+
 
+
x
+
c
 
2
 
2
z
+
)
 +
3
 +
/
 
2
 
2
 +
3
 
.
 
.
x
+
a
4
+
3
  +y
+
  +b
 +
3
 +
+c
 +
3
 +
≥3(
 +
3
 +
a
 +
2
 +
+b
 
2
 
2
  +z
+
  +c
 
2
 
2
 
x
 
4
 
 
   
 
   
 
 
 +
)
 +
3/2
 
  =  
 
  =  
x
+
3
6
+
  +x
+
   
 +
(a
 
2
 
2
  y
+
  +b
 
2
 
2
  +x
+
  +c
2
 
z
 
 
2
 
2
   
+
  )
x
+
3/2
6
 
 
   
 
   
 
 
 
  .
 
  .
Thus,
+
Squaring both sides (all positive) gives:
  
LHS
+
(
=
+
a
+
3
x
 
4
 
x
 
4
 
 
+
 
+
y
+
b
2
+
3
 
+
 
+
z
+
c
 +
3
 +
)
 +
2
 +
 +
(
 +
a
 
2
 
2
=
 
 
x
 
6
 
x
 
6
 
 
+
 
+
x
+
b
2
 
y
 
 
2
 
2
 
+
 
+
x
+
c
 
2
 
2
z
+
)
 +
3
 +
3
 +
.
 +
(a
 +
3
 +
+b
 +
3
 +
+c
 +
3
 +
)
 
2
 
2
.
+
LHS=∑
+
3
x
+
(a
4
 
+y
 
 
2
 
2
  +z
+
  +b
 
2
 
2
+
  +c
x
 
4
 
 
 
=∑
 
x
 
6
 
  +x
 
2
 
y
 
2
 
+x
 
2
 
z
 
 
2
 
2
   
+
  )
x
+
3
6
 
 
   
 
   
 
 
 
  .
 
  .
By the Cauchy–Schwarz inequality,
+
That is,
  
 
(
 
(
+
a
x
+
2
6
 
x
 
6
 
 
+
 
+
x
+
b
2
 
y
 
 
2
 
2
 
+
 
+
x
+
c
2
 
z
 
 
2
 
2
 
)
 
)
 +
3
 +
 +
3
 
(
 
(
+
a
(
+
3
x
 
6
 
 
+
 
+
x
+
b
2
+
3
y
 
2
 
 
+
 
+
x
+
c
2
 
z
 
2
 
)
 
)
 
 
(
 
 
x
 
 
3
 
3
 
)
 
)
 
2
 
2
 
.
 
.
(
+
(5)
x
+
(a
6
 
+x
 
 
2
 
2
  y
+
  +b
 
2
 
2
  +x
+
  +c
 
2
 
2
  z
+
  )
2
+
3
+
  ≤3(a
x
+
3
6
+
  +b
   
+
3
+
  +c
)(∑(x
 
6
 
  +x
 
2
 
y
 
2
 
  +x
 
2
 
z
 
2
 
))≥(∑x
 
 
3
 
3
 
  )  
 
  )  
 
2
 
2
  .
+
  .(5)
But
+
On the other hand, since
 
+
a
+
3
(
 
x
 
6
 
 
+
 
+
x
+
b
2
+
3
y
 
2
 
 
+
 
+
x
+
c
2
+
3
z
+
2
+
3
)
+
a
=
+
3
+
+b
x
+
3
6
+
+c
 +
3
 +
≥3, we have
 +
 
 +
3
 +
(
 +
a
 +
3
 
+
 
+
+
b
x
+
3
2
 
y
 
2
 
 
+
 
+
+
c
x
+
3
 +
)
 
2
 
2
z
+
2
+
(
=
+
a
+
3
x
+
+
6
+
b
 +
3
 +
+
 +
c
 +
3
 +
)
 +
3
 +
,
 +
because
 +
(
 +
a
 +
3
 +
+
 +
b
 +
3
 +
+
 +
c
 +
3
 +
)
 +
3
 +
 +
3
 +
(
 +
a
 +
3
 +
+
 +
b
 +
3
 
+
 
+
 +
c
 +
3
 +
)
 
2
 
2
+
x
+
a
2
 
y
 
2
 
.
 
∑(x
 
6
 
+x
 
2
 
y
 
2
 
+x
 
2
 
z
 
2
 
)=∑x
 
6
 
+∑x
 
2
 
y
 
2
 
+∑x
 
2
 
z
 
2
 
=∑x
 
6
 
+2∑x
 
2
 
y
 
2
 
.
 
Hence,
 
 
 
LHS
 
 
(
 
x
 
 
3
 
3
 
+
 
+
y
+
b
 
3
 
3
 
+
 
+
z
+
c
 +
3
 +
 +
3.
 +
3(a
 +
3
 +
+b
 +
3
 +
+c
 
3
 
3
)
+
)  
 
2
 
2
x
+
≤(a
6
+
3
+
+
+b
y
+
3
6
+
+c
+
+
3
z
+
)
6
+
3
+
+
,because(a
2
+
3
(
+
+b
x
+
3
2
+
+c
y
+
3
 +
)
 +
3
 +
≥3(a
 +
3
 +
+b
 +
3
 +
+c
 +
3
 +
)
 +
2
 +
⇔a
 +
3
 +
+b
 +
3
 +
+c
 +
3
 +
≥3.
 +
Combining with (5), we get:
 +
 
 +
(
 +
a
 
2
 
2
 
+
 
+
y
+
b
2
 
z
 
 
2
 
2
 
+
 
+
z
+
c
2
 
x
 
 
2
 
2
 
)
 
)
.
+
3
LHS≥
+
x
+
(
6
+
a
+y
+
3
6
+
+
+z
+
b
6
+
3
+2(x
+
+
 +
c
 +
3
 +
)
 +
3
 +
,
 +
(a
 
2
 
2
  y
+
  +b
 
2
 
2
  +y
+
  +c
 
2
 
2
  z
+
  )
2
+
3
  +z
+
  (a
2
 
x
 
2
 
)
 
(x
 
 
3
 
3
  +y
+
  +b
 
3
 
3
  +z
+
  +c
 
3
 
3
 
  )  
 
  )  
 +
3
 +
,
 +
so taking cube roots yields
 +
a
 
2
 
2
+
+
+
b
.
+
2
We now show that this expression is at least 1. That is, we need
+
+
 
+
c
(
+
2
x
+
 +
a
 
3
 
3
 
+
 
+
y
+
b
 
3
 
3
 
+
 
+
z
+
c
 
3
 
3
)
+
a
 +
2
 +
+b
 +
2
 +
+c
 
2
 
2
+
≤a
 +
3
 +
+b
 +
3
 +
+c
 +
3
 +
, as desired.
 +
 
 +
Therefore, from (3) we obtain
 +
 
 +
 +
x
 +
4
 
x
 
x
6
+
4
 
+
 
+
 
y
 
y
6
+
2
 
+
 
+
 
z
 
z
6
 
+
 
 
2
 
2
(
+
 +
1.
 +
(6)
 +
 +
x
 +
4
 +
+y
 +
2
 +
+z
 +
2
 +
 +
x
 +
4
 +
 +
 +
≥1.(6)
 +
Combining (2) and (6), we have
 +
 
 +
 +
x
 +
4
 
x
 
x
 +
4
 +
+
 +
y
 
2
 
2
y
+
+
 +
z
 
2
 
2
 +
 +
1
 +
 +
 +
x
 +
x
 +
4
 
+
 
+
 
y
 
y
2
 
z
 
 
2
 
2
 
+
 
+
 
z
 
z
 
2
 
2
x
+
,
2
+
)
+
x  
.
+
4
(x  
 
3
 
 
  +y  
 
  +y  
3
 
+z
 
3
 
)
 
 
2
 
2
≥x
 
6
 
+y
 
6
 
 
  +z  
 
  +z  
6
 
+2(x
 
2
 
y
 
 
2
 
2
 +
 +
x
 +
4
 +
 +
 +
≥1≥∑
 +
x
 +
4
 
  +y  
 
  +y  
2
 
z
 
 
2
 
2
 
  +z  
 
  +z  
 
2
 
2
  x  
+
   
2
+
x
  ).
+
Expanding the left-hand side:
+
  ,
 +
which proves (1). Hence, the original inequality holds.
  
(
+
Equality occurs when
 
x
 
x
3
+
=
+
 
 
y
 
y
3
+
=
+
 
 
z
 
z
3
 
)
 
2
 
 
=
 
=
x
+
1
6
+
x=y=z=1.
+
 
y
 
6
 
+
 
z
 
6
 
+
 
2
 
(
 
x
 
3
 
y
 
3
 
+
 
x
 
3
 
z
 
3
 
+
 
y
 
3
 
z
 
3
 
)
 
.
 
(x
 
3
 
+y
 
3
 
+z
 
3
 
)
 
2
 
=x
 
6
 
+y
 
6
 
+z
 
6
 
+2(x
 
3
 
y
 
3
 
+x
 
3
 
z
 
3
 
+y
 
3
 
z
 
3
 
).
 
So the inequality becomes
 
 
 
x
 
3
 
y
 
3
 
+
 
x
 
3
 
z
 
3
 
+
 
y
 
3
 
z
 
3
 
 
x
 
2
 
y
 
2
 
+
 
y
 
2
 
z
 
2
 
+
 
z
 
2
 
x
 
2
 
.
 
x
 
3
 
y
 
3
 
+x
 
3
 
z
 
3
 
+y
 
3
 
z
 
3
 
≥x
 
2
 
y
 
2
 
+y
 
2
 
z
 
2
 
+z
 
2
 
x
 
2
 
.
 
Let <math>a = xy</math>, <math>b = yz</math>, <math>c = zx</math>. Then the inequality is equivalent to
 
 
 
a
 
3
 
+
 
b
 
3
 
+
 
c
 
3
 
 
a
 
2
 
+
 
b
 
2
 
+
 
c
 
2
 
.
 
a
 
3
 
+b
 
3
 
+c
 
3
 
≥a
 
2
 
+b
 
2
 
+c
 
2
 
.
 
Note that <math>abc = x^2y^2z^2 \geq 1</math> since <math>xyz \geq 1</math>. Also, by the AM–GM inequality, <math>a^3 + b^3 + c^3 \geq 3\sqrt[3]{a^3b^3c^3} = 3abc \geq 3</math>.
 
 
 
Now, by the power mean inequality, we have
 
 
 
(
 
a
 
3
 
+
 
b
 
3
 
+
 
c
 
3
 
3
 
)
 
1
 
/
 
3
 
 
(
 
a
 
2
 
+
 
b
 
2
 
+
 
c
 
2
 
3
 
)
 
1
 
/
 
2
 
,
 
(
 
3
 
a
 
3
 
+b
 
3
 
+c
 
3
 
 
 
)
 
1/3
 
≥(
 
3
 
a
 
2
 
+b
 
2
 
+c
 
2
 
 
 
)
 
1/2
 
,
 
which implies
 
 
 
(
 
a
 
3
 
+
 
b
 
3
 
+
 
c
 
3
 
)
 
2
 
 
1
 
3
 
(
 
a
 
2
 
+
 
b
 
2
 
+
 
c
 
2
 
)
 
3
 
.
 
(a
 
3
 
+b
 
3
 
+c
 
3
 
)
 
2
 
 
3
 
1
 
 
(a
 
2
 
+b
 
2
 
+c
 
2
 
)
 
3
 
.
 
That is,
 
 
 
(
 
a
 
2
 
+
 
b
 
2
 
+
 
c
 
2
 
)
 
3
 
 
3
 
(
 
a
 
3
 
+
 
b
 
3
 
+
 
c
 
3
 
)
 
2
 
.
 
(a
 
2
 
+b
 
2
 
+c
 
2
 
)
 
3
 
≤3(a
 
3
 
+b
 
3
 
+c
 
3
 
)
 
2
 
.
 
Since <math>a^3 + b^3 + c^3 \geq 3</math>, we have
 
 
 
3
 
(
 
a
 
3
 
+
 
b
 
3
 
+
 
c
 
3
 
)
 
2
 
 
(
 
a
 
3
 
+
 
b
 
3
 
+
 
c
 
3
 
)
 
3
 
.
 
3(a
 
3
 
+b
 
3
 
+c
 
3
 
)
 
2
 
≤(a
 
3
 
+b
 
3
 
+c
 
3
 
)
 
3
 
.
 
Combining, we get
 
 
 
(
 
a
 
2
 
+
 
b
 
2
 
+
 
c
 
2
 
)
 
3
 
 
(
 
a
 
3
 
+
 
b
 
3
 
+
 
c
 
3
 
)
 
3
 
,
 
(a
 
2
 
+b
 
2
 
+c
 
2
 
)
 
3
 
≤(a
 
3
 
+b
 
3
 
+c
 
3
 
)
 
3
 
,
 
so taking cube roots yields
 
 
 
a
 
2
 
+
 
b
 
2
 
+
 
c
 
2
 
 
a
 
3
 
+
 
b
 
3
 
+
 
c
 
3
 
,
 
a
 
2
 
+b
 
2
 
+c
 
2
 
≤a
 
3
 
+b
 
3
 
+c
 
3
 
,
 
as desired. Therefore, LHS <math>\geq 1</math>.
 
 
 
Since we have shown that LHS <math>\geq 1</math> and RHS <math>\leq 1</math>, it follows that
 
 
 
x
 
4
 
x
 
4
 
+
 
y
 
2
 
+
 
z
 
2
 
+
 
y
 
4
 
y
 
4
 
+
 
z
 
2
 
+
 
x
 
2
 
+
 
z
 
4
 
z
 
4
 
+
 
x
 
2
 
+
 
y
 
2
 
 
x
 
x
 
4
 
+
 
y
 
2
 
+
 
z
 
2
 
+
 
y
 
y
 
4
 
+
 
z
 
2
 
+
 
x
 
2
 
+
 
z
 
z
 
4
 
+
 
x
 
2
 
+
 
y
 
2
 
,
 
x
 
4
 
+y
 
2
 
+z
 
2
 
 
x
 
4
 
 
 
+
 
y
 
4
 
+z
 
2
 
+x
 
2
 
 
y
 
4
 
 
 
+
 
z
 
4
 
+x
 
2
 
+y
 
2
 
 
z
 
4
 
 
 
 
x
 
4
 
+y
 
2
 
+z
 
2
 
 
x
 
 
+
 
y
 
4
 
+z
 
2
 
+x
 
2
 
 
y
 
 
+
 
z
 
4
 
+x
 
2
 
+y
 
2
 
 
z
 
 
,
 
and hence the original inequality holds.
 
 
 
Equality occurs when <math>x = y = z = 1</math>.
 
  
 
\end{document}
 
\end{document}

Latest revision as of 03:37, 4 September 2025

\documentclass{article} \usepackage{amsmath} \usepackage{amssymb} \begin{document}

\section*{Problem} Let x , y , z x,y,z be positive real numbers such that x y z ≥ 1 xyz≥1. Prove that:

x 5 − x 2 x 5 + y 2 + z 2 + y 5 − y 2 y 5 + z 2 + x 2 + z 5 − z 2 z 5 + x 2 + y 2 ≥ 0. x 5

+y 

2

+z 

2

x 5

−x 

2

+ 

y 5

+z 

2

+x 

2

y 5

−y 

2

+ 

z 5

+x 

2

+y 

2

z 5

−z 

2

≥0.

\section*{Solution} We first show that for any positive real numbers x , y , z x,y,z, we have

x 5 − x 2 x 5 + y 2 + z 2 ≥ x 4 − x x 4 + y 2 + z 2 . x 5

+y 

2

+z 

2

x 5

−x 

2

x 4

+y 

2

+z 

2

x 4

−x

.

Indeed, consider the difference: \begin{align*} &\frac{x^5 - x^2}{x^5 + y^2 + z^2} - \frac{x^4 - x}{x^4 + y^2 + z^2} \ &= \frac{(x^5 - x^2)(x^4 + y^2 + z^2) - (x^4 - x)(x^5 + y^2 + z^2)}{(x^5 + y^2 + z^2)(x^4 + y^2 + z^2)}. \end{align*} The numerator simplifies as follows: \begin{align*} &(x^5 - x^2)(x^4 + y^2 + z^2) - (x^4 - x)(x^5 + y^2 + z^2) \ &= x^5x^4 + x^5(y^2+z^2) - x^2x^4 - x^2(y^2+z^2) - x^4x^5 - x^4(y^2+z^2) + x x^5 + x(y^2+z^2) \ &= x^5(y^2+z^2) - x^2(y^2+z^2) - x^4(y^2+z^2) + x(y^2+z^2) \ &= (y^2+z^2)(x^5 - x^4 - x^2 + x) \ &= (y^2+z^2)x(x^4 - x^3 - x + 1) \quad \text{(but wait, check: actually, } x^5 - x^4 - x^2 + x = x(x^4 - x^3 - x + 1) \text{? That doesn't factor nicely)}. \end{align*} Alternatively, note that:

x 5 − x 2 = x 2 ( x 3 − 1 ) = x 2 ( x − 1 ) ( x 2 + x + 1 ) , x 4 − x = x ( x 3 − 1 ) = x ( x − 1 ) ( x 2 + x + 1 ) . x 5

−x 

2

=x 

2

(x 

3

−1)=x 

2

(x−1)(x 

2

+x+1),x 

4

−x=x(x 

3

−1)=x(x−1)(x 

2

+x+1).

So then: \begin{align*} &\frac{x^5 - x^2}{x^5 + y^2 + z^2} - \frac{x^4 - x}{x^4 + y^2 + z^2} \ &= \frac{x^2(x-1)(x^2+x+1)}{x^5 + y^2 + z^2} - \frac{x(x-1)(x^2+x+1)}{x^4 + y^2 + z^2} \ &= x(x-1)(x^2+x+1) \left( \frac{x}{x^5 + y^2 + z^2} - \frac{1}{x^4 + y^2 + z^2} \right). \end{align*} Now, combine the terms inside the parentheses:

x x 5 + y 2 + z 2 − 1 x 4 + y 2 + z 2 = x ( x 4 + y 2 + z 2 ) − ( x 5 + y 2 + z 2 ) ( x 5 + y 2 + z 2 ) ( x 4 + y 2 + z 2 ) = ( x 5 + x ( y 2 + z 2 ) ) − ( x 5 + y 2 + z 2 ) ( x 5 + y 2 + z 2 ) ( x 4 + y 2 + z 2 ) = ( x − 1 ) ( y 2 + z 2 ) ( x 5 + y 2 + z 2 ) ( x 4 + y 2 + z 2 ) . x 5

+y 

2

+z 

2

x ​

x 4

+y 

2

+z 

2

1 ​

= 

(x 5

+y 

2

+z 

2

)(x 

4

+y 

2

+z 

2

)

x(x 4

+y 

2

+z 

2

)−(x 

5

+y 

2

+z 

2

)

= 

(x 5

+y 

2

+z 

2

)(x 

4

+y 

2

+z 

2

)

(x 5

+x(y 

2

+z 

2

))−(x 

5

+y 

2

+z 

2

)

= 

(x 5

+y 

2

+z 

2

)(x 

4

+y 

2

+z 

2

)

(x−1)(y 2

+z 

2

)

.

Thus, the entire difference becomes:

x ( x − 1 ) ( x 2 + x + 1 ) ⋅ ( x − 1 ) ( y 2 + z 2 ) ( x 5 + y 2 + z 2 ) ( x 4 + y 2 + z 2 ) = x ( x − 1 ) 2 ( x 2 + x + 1 ) ( y 2 + z 2 ) ( x 5 + y 2 + z 2 ) ( x 4 + y 2 + z 2 ) ≥ 0. x(x−1)(x 2

+x+1)⋅ 

(x 5

+y 

2

+z 

2

)(x 

4

+y 

2

+z 

2

)

(x−1)(y 2

+z 

2

)

= 

(x 5

+y 

2

+z 

2

)(x 

4

+y 

2

+z 

2

)

x(x−1) 2

(x 

2

+x+1)(y 

2

+z 

2

)

≥0.

Hence,

x 5 − x 2 x 5 + y 2 + z 2 ≥ x 4 − x x 4 + y 2 + z 2 . x 5

+y 

2

+z 

2

x 5

−x 

2

x 4

+y 

2

+z 

2

x 4

−x

.

Similarly, we have the analogous inequalities for y y and z z. Therefore, it suffices to prove that

x 4 − x x 4 + y 2 + z 2 + y 4 − y y 4 + z 2 + x 2 + z 4 − z z 4 + x 2 + y 2 ≥ 0 , x 4

+y 

2

+z 

2

x 4

−x

+ 

y 4

+z 

2

+x 

2

y 4

−y

+ 

z 4

+x 

2

+y 

2

z 4

−z

≥0,

or equivalently,

x 4 x 4 + y 2 + z 2 + y 4 y 4 + z 2 + x 2 + z 4 z 4 + x 2 + y 2 ≥ x x 4 + y 2 + z 2 + y y 4 + z 2 + x 2 + z z 4 + x 2 + y 2 . (1) x 4

+y 

2

+z 

2

x 4

+ 

y 4

+z 

2

+x 

2

y 4

+ 

z 4

+x 

2

+y 

2

z 4

x 4

+y 

2

+z 

2

x ​

+ 

y 4

+z 

2

+x 

2

y ​

+ 

z 4

+x 

2

+y 

2

z ​

.(1)

Let t = x + y + z t=x+y+z. Since x , y , z > 0 x,y,z>0 and x y z ≥ 1 xyz≥1, by AM–GM we have t ≥ 3 x y z 3 ≥ 3 t≥3 3

xyz ​

≥3. Also, note that 

x y + y z + z x ≤ t 2 3 xy+yz+zx≤ 3 t 2

 and 

x 2 + y 2 + z 2 ≥ t 2 3 x 2

+y 

2

+z 

2

3 t 2

.

Now, we estimate the right-hand side of (1). By the Cauchy–Schwarz inequality, we have:

( x 2 + y 2 + z 2 ) 2 ≤ ( x 4 + y 2 + z 2 ) ( 1 + y 2 + z 2 ) , (x 2

+y 

2

+z 

2

) 

2

≤(x 

4

+y 

2

+z 

2

)(1+y 

2

+z 

2

),

since

( x 2 + y 2 + z 2 ) 2 = ( x 2 ⋅ 1 + y ⋅ y + z ⋅ z ) 2 ≤ ( x 4 + y 2 + z 2 ) ( 1 2 + y 2 + z 2 ) = ( x 4 + y 2 + z 2 ) ( 1 + y 2 + z 2 ) . (x 2

+y 

2

+z 

2

) 

2

=(x 

2

⋅1+y⋅y+z⋅z) 

2

≤(x 

4

+y 

2

+z 

2

)(1 

2

+y 

2

+z 

2

)=(x 

4

+y 

2

+z 

2

)(1+y 

2

+z 

2

).

It follows that

1 x 4 + y 2 + z 2 ≤ 1 + y 2 + z 2 ( x 2 + y 2 + z 2 ) 2 , x 4

+y 

2

+z 

2

1 ​

(x 2

+y 

2

+z 

2

) 

2

1+y 2

+z 

2

,

and multiplying by x x (which is positive) gives:

x x 4 + y 2 + z 2 ≤ x ( 1 + y 2 + z 2 ) ( x 2 + y 2 + z 2 ) 2 . x 4

+y 

2

+z 

2

x ​

(x 2

+y 

2

+z 

2

) 

2

x(1+y 2

+z 

2

)

.

Similarly,

y y 4 + z 2 + x 2 ≤ y ( 1 + z 2 + x 2 ) ( x 2 + y 2 + z 2 ) 2 , z z 4 + x 2 + y 2 ≤ z ( 1 + x 2 + y 2 ) ( x 2 + y 2 + z 2 ) 2 . y 4

+z 

2

+x 

2

y ​

(x 2

+y 

2

+z 

2

) 

2

y(1+z 2

+x 

2

)

, 

z 4

+x 

2

+y 

2

z ​

(x 2

+y 

2

+z 

2

) 

2

z(1+x 2

+y 

2

)

.

Summing these, we obtain:

∑ x x 4 + y 2 + z 2 ≤ 1 ( x 2 + y 2 + z 2 ) 2 [ ∑ x + ∑ x ( y 2 + z 2 ) ] . ∑ x 4

+y 

2

+z 

2

x ​

(x 2

+y 

2

+z 

2

) 

2

1 ​

[∑x+∑x(y 

2

+z 

2

)].

Now, note that

∑ x ( y 2 + z 2 ) = ∑ ( x y 2 + x z 2 ) = ( x y 2 + x z 2 ) + ( y z 2 + y x 2 ) + ( z x 2 + z y 2 ) = ( x + y + z ) ( x y + y z + z x ) − 3 x y z . ∑x(y 2

+z 

2

)=∑(xy 

2

+xz 

2

)=(xy 

2

+xz 

2

)+(yz 

2

+yx 

2

)+(zx 

2

+zy 

2

)=(x+y+z)(xy+yz+zx)−3xyz.

Thus,

∑ x x 4 + y 2 + z 2 ≤ t + t ( x y + y z + z x ) − 3 x y z ( x 2 + y 2 + z 2 ) 2 . ∑ x 4

+y 

2

+z 

2

x ​

(x 2

+y 

2

+z 

2

) 

2

t+t(xy+yz+zx)−3xyz ​

.

Since x y z ≥ 1 xyz≥1, we have − 3 x y z ≤ − 3 −3xyz≤−3. Also, x y + y z + z x ≤ t 2 3 xy+yz+zx≤ 3 t 2

 and 

x 2 + y 2 + z 2 ≥ t 2 3 x 2

+y 

2

+z 

2

3 t 2

, so

( x 2 + y 2 + z 2 ) 2 ≥ ( t 2 3 ) 2 = t 4 9 . (x 2

+y 

2

+z 

2

) 

2

≥( 

3 t 2

) 

2

= 

9 t 4

.

Therefore,

∑ x x 4 + y 2 + z 2 ≤ t + t ⋅ t 2 3 − 3 ( t 2 / 3 ) 2 = t + t 3 3 − 3 t 4 / 9 = 9 ( t + t 3 3 − 3 ) t 4 = 9 t + 3 t 3 − 27 t 4 = 3 t 3 + 9 t − 27 t 4 . ∑ x 4

+y 

2

+z 

2

x ​

(t 2

/3) 

2

t+t⋅ 3 t 2

−3

= 

t 4

/9

t+ 3 t 3

−3

= 

t 4

9(t+ 3 t 3

−3)

= 

t 4

9t+3t 3

−27

= 

t 4

3t 3

+9t−27

.

We now show that

3 t 3 + 9 t − 27 t 4 ≤ 1. t 4

3t 3

+9t−27

≤1.

This is equivalent to

3 t 3 + 9 t − 27 ≤ t 4 ⇔ t 4 − 3 t 3 − 9 t + 27 ≥ 0. 3t 3

+9t−27≤t 

4

⇔t 

4

−3t 

3

−9t+27≥0.

Factor the left-hand side:

t 4 − 3 t 3 − 9 t + 27 = ( t − 3 ) ( t 3 − 9 ) . t 4

−3t 

3

−9t+27=(t−3)(t 

3

−9).

For t ≥ 3 t≥3, we have t − 3 ≥ 0 t−3≥0 and t 3 − 9 ≥ 18 t 3

−9≥18, so indeed 

( t − 3 ) ( t 3 − 9 ) ≥ 0 (t−3)(t 3

−9)≥0. Hence,

∑ x x 4 + y 2 + z 2 ≤ 1. (2) ∑ x 4

+y 

2

+z 

2

x ​

≤1.(2)

Now, we estimate the left-hand side of (1). Note that

x 4 x 4 + y 2 + z 2 = x 6 x 6 + x 2 y 2 + x 2 z 2 , x 4

+y 

2

+z 

2

x 4

= 

x 6

+x 

2

y 

2

+x 

2

z 

2

x 6

,

and similarly for the other terms. So,

∑ x 4 x 4 + y 2 + z 2 = ∑ x 6 x 6 + x 2 y 2 + x 2 z 2 . ∑ x 4

+y 

2

+z 

2

x 4

=∑ 

x 6

+x 

2

y 

2

+x 

2

z 

2

x 6

.

By the Cauchy–Schwarz inequality (Titu's lemma), we have:

∑ x 6 x 6 + x 2 y 2 + x 2 z 2 ≥ ( x 3 + y 3 + z 3 ) 2 ∑ ( x 6 + x 2 y 2 + x 2 z 2 ) = ( x 3 + y 3 + z 3 ) 2 x 6 + y 6 + z 6 + 2 ( x 2 y 2 + y 2 z 2 + z 2 x 2 ) . (3) ∑ x 6

+x 

2

y 

2

+x 

2

z 

2

x 6

∑(x 6

+x 

2

y 

2

+x 

2

z 

2

)

(x 3

+y 

3

+z 

3

) 

2

= 

x 6

+y 

6

+z 

6

+2(x 

2

y 

2

+y 

2

z 

2

+z 

2

x 

2

)

(x 3

+y 

3

+z 

3

) 

2

.(3)

We claim that

( x 3 + y 3 + z 3 ) 2 ≥ x 6 + y 6 + z 6 + 2 ( x 2 y 2 + y 2 z 2 + z 2 x 2 ) . (x 3

+y 

3

+z 

3

) 

2

≥x 

6

+y 

6

+z 

6

+2(x 

2

y 

2

+y 

2

z 

2

+z 

2

x 

2

).

Expanding the left-hand side:

( x 3 + y 3 + z 3 ) 2 = x 6 + y 6 + z 6 + 2 ( x 3 y 3 + y 3 z 3 + z 3 x 3 ) . (x 3

+y 

3

+z 

3

) 

2

=x 

6

+y 

6

+z 

6

+2(x 

3

y 

3

+y 

3

z 

3

+z 

3

x 

3

).

Thus, the inequality is equivalent to

x 3 y 3 + y 3 z 3 + z 3 x 3 ≥ x 2 y 2 + y 2 z 2 + z 2 x 2 . (4) x 3

y 

3

+y 

3

z 

3

+z 

3

x 

3

≥x 

2

y 

2

+y 

2

z 

2

+z 

2

x 

2

.(4)

Let a = x y a=xy, b = y z b=yz, c = z x c=zx. Then a b c = x 2 y 2 z 2 ≥ 1 abc=x 2

y 

2

z 

2

≥1, and inequality (4) becomes:

a 3 + b 3 + c 3 ≥ a 2 + b 2 + c 2 . a 3

+b 

3

+c 

3

≥a 

2

+b 

2

+c 

2

.

We now prove this. By the AM–GM inequality, a 3 + b 3 + c 3 ≥ 3 a b c ≥ 3 a 3

+b 

3

+c 

3

≥3abc≥3. Also, by the power mean inequality, we have:

( a 3 + b 3 + c 3 3 ) 1 / 3 ≥ ( a 2 + b 2 + c 2 3 ) 1 / 2 , ( 3 a 3

+b 

3

+c 

3

) 

1/3

≥( 

3 a 2

+b 

2

+c 

2

) 

1/2

,

which implies

a 3 + b 3 + c 3 ≥ 3 ( a 2 + b 2 + c 2 3 ) 3 / 2 = ( a 2 + b 2 + c 2 ) 3 / 2 3 . a 3

+b 

3

+c 

3

≥3( 

3 a 2

+b 

2

+c 

2

) 

3/2

= 

3 ​

(a 2

+b 

2

+c 

2

) 

3/2

.

Squaring both sides (all positive) gives:

( a 3 + b 3 + c 3 ) 2 ≥ ( a 2 + b 2 + c 2 ) 3 3 . (a 3

+b 

3

+c 

3

) 

2

3 (a 2

+b 

2

+c 

2

) 

3

.

That is,

( a 2 + b 2 + c 2 ) 3 ≤ 3 ( a 3 + b 3 + c 3 ) 2 . (5) (a 2

+b 

2

+c 

2

) 

3

≤3(a 

3

+b 

3

+c 

3

) 

2

.(5)

On the other hand, since a 3 + b 3 + c 3 ≥ 3 a 3

+b 

3

+c 

3

≥3, we have

3 ( a 3 + b 3 + c 3 ) 2 ≤ ( a 3 + b 3 + c 3 ) 3 , because ( a 3 + b 3 + c 3 ) 3 ≥ 3 ( a 3 + b 3 + c 3 ) 2 ⇔ a 3 + b 3 + c 3 ≥ 3. 3(a 3

+b 

3

+c 

3

) 

2

≤(a 

3

+b 

3

+c 

3

) 

3

,because(a 

3

+b 

3

+c 

3

) 

3

≥3(a 

3

+b 

3

+c 

3

) 

2

⇔a 

3

+b 

3

+c 

3

≥3.

Combining with (5), we get:

( a 2 + b 2 + c 2 ) 3 ≤ ( a 3 + b 3 + c 3 ) 3 , (a 2

+b 

2

+c 

2

) 

3

≤(a 

3

+b 

3

+c 

3

) 

3

,

so taking cube roots yields a 2 + b 2 + c 2 ≤ a 3 + b 3 + c 3 a 2

+b 

2

+c 

2

≤a 

3

+b 

3

+c 

3

, as desired.

Therefore, from (3) we obtain

∑ x 4 x 4 + y 2 + z 2 ≥ 1. (6) ∑ x 4

+y 

2

+z 

2

x 4

≥1.(6)

Combining (2) and (6), we have

∑ x 4 x 4 + y 2 + z 2 ≥ 1 ≥ ∑ x x 4 + y 2 + z 2 , ∑ x 4

+y 

2

+z 

2

x 4

≥1≥∑ 

x 4

+y 

2

+z 

2

x ​

,

which proves (1). Hence, the original inequality holds.

Equality occurs when x = y = z = 1 x=y=z=1.

\end{document}